You are on page 1of 91

EMLE exams

There is an answered version of this file

Index
19 November 2023 (2nd trial exam) ......................................................................................................... 2
18 November 2023 (1st trial exam) ........................................................................................................ 24
6 June 2023 real exam ............................................................................................................................. 46
11 June 2023 3rd real exam .................................................................................................................... 67

Class 190 KA - Kasr Alainy - Cairo University

1
19 November 2023 (2nd trial exam)

1) A 47-year-old man came to ER complaining of acute abdominal pain, distension, vomiting and
constipation for 4 days. The patient gave a history of surgery 5 years ago for excision of abdom-
inal mass which was of benign nature on histopathological examination. Also, there is a history
of similar manifestations that occurred twice 1 & 3 years after operation, X ray, U/S and CT were
done which do not detect any pathology except dilated bowel loops.Which of the following is the
etiology of these symptoms?
Recurrence of the tumour
Bowel ischernia
Adhesive intestinal obstruction
Concer colon
Concer stomach

2) A 59-year-old man with a known history of chronic obstructive pulmonary disease presents with
worsening dyspnea. On examination, he is afebrile. His breath sounds are decreased bilaterally.
He is noted to have jugular venous distension (JVD) and 2+ pitting edema of the lower extremi-
ties. Which of the following is the most likely cause of worsening his condition?
Bronchogenic carcinoma
COPD exacerbation
cor pulmonale
Pneumonia
Pneumothorax

3) A 63-year-old woman is admitted to the hospital with severe abdominal pain of 3-hour duration.
Abdominal examination reveals board-like rigidity, guarding, and rebound tenderness. Her blood
pressure is 90/50 mm Hg, pulse 110 bpm (beats per minute), and respiratory rate is 30 breaths
per minute. After a thorough history and physical examination, and initiation of fluid resuscita-
tion, which of the following diagnostic studies should be performed?
Supine abdominal x-rays
Upright chest x-ray
Gastrograffin swallow
Computerized axial tornography (CAT) scan of the abdomen
Abdominal sonogram

4) A 2-year-old child was brought to the Pediatric outpatient clinic by his parents. They stated that
two months before, his lips and tongue became blue whenever he played. On examination, he
looked ill and smaller than expected for age, had bluish discoloration of the lips, tongue and fin-
ger nails that showed early clubbing. Respiratory rate: 30/min. Heart examination showed dif-
fuse precordial pulsations more marked on the left parasternal area. He improved by squatting.
Which of the following is the most probable diagnosis of this case?
Atrial septal defect
DiGeorge syndrome
Eisenmenger syndrome
Fallot’s tetralogy
Transposition of great vessels

2
5) A 73-year-old man undergoes an elective total gastrectomy via an upper midline abdominal Inci-
sion. On the first postoperative day, he is confused and disorientated in time, place and person,
and pulling at his catheter and drains. On examination, his temperature is 38° C, 02 saturation is
87% on 21 02 Which of the following is the most likely cause of the patient’s confusion?
Chest infection
Cerebrovascular accident (CVA)
Hypoxia
CO2 retention
Opiate toxicity

6) A 61-year-old man complained of progressive yellowish discoloration of the sclera, itching and
vague abdominal pain Carcinoembryonic antigen (CEA) was 523 ng/mL. (0-24 ng/mL), CA 19-9
was 450 Unit/mL (0-37 Unit/mL) and serum bilirubin was 12 mg/dl (0.3-1.2 mg/dl). Ultrasound
study was unremarkable and CT scan of the abdomen revealed bulky pancreas and multiple
scattered hepatic metastatic nodules. Which of the following is the best management for this
patient?
ERCP stenting
Pancreaticoduodenectomy
Pancreaticoduodenectomy hepatic resection
Radiotherapy only
Total pancreatectomy

7) A 32-year-old man presented with low back pain for 3 months. The pain is worse on awakening
and gets better on movements. On examination, there was limitation in forward and lateral
bending of the trunk. What is the most useful test for diagnosing the cause of his back pain?
Bone mineral density by DEXA scan
CT scan of lumbar spines and pelvis
HLA B27 detection
Nerve conduction studies for lower limbs
Plain X ray lumbosacral spines

8) A 28-year-old G2PO at 39 weeks is in early labor. She has no antenatal care visits for 3 months.
Her blood pressure at admission was 180/120 mm Hg and she was transferred to the intensive
care to continue her management. Her cervix is 2 cm dilated and 90% effaced, with contractions
every 4 to 5 minutes. The fetal heart tones are reassuring. Her nurse steps out for a moment and
returns to find her having a seizure. Which of the following complications puts this patient most
at risk for mortality?
hypovolemic shock
uremia
congestive heart failure
severe infection
cerebral hemorrhage

9) A33-year-old G 2 PO+1 at 15 week’s gestation has a history of chronic hypertension. She presents
for her first prenatal visit. Vital signs. T 98°F (36.5°C), P 66 beats/min, R 16 breaths/min, BP
146/98 mmHg. She stopped taking her BP medication 6 weeks ago when she found out that she
3
was pregnant. Of the following medications, which is the best choice for treating chronic hyper-
tension in pregnancy?
Hydralazine
Hydrochlorothiazide
Labetalol
Lisinopril
Atenolol

10) A 23-year-old man came to the colorectal outpatient clinic complaining of acute sharp pain in
anus after defecation. This pain persists for hours after evacuation. There is some bright red
blood on the toilet paper when he wipes. There is increasing constipation. No weight loss. On
examination, anus and recturn are externally normal. However, digital examination could not be
performed because of severe discomfort and sphincter spasm. Which of the following is the most
likely diagnosis?
Anal squamous cell carcinoma
Acute anal fissure
Grade IV haemorrhoids
Diverticular disease
Anal fistula

11) A 70-year-old woman presented with progressive weakness and fatigue about a month earli-
er, and she no longer felt well enough to do her housework or take her daily walk. Although her
breathing was normal at rest, she was too short of breath on exertion. On physical examination,
she is found to be anemic and has dyspepsia and peripheral neuropathy. Laboratory studies re-
vealed:• RBCS: 3.5 million/mm3. Hemoglobin: 9 gm/dl. Hematocrit 29 % (N. 35.5 to 44.9 %) RBCS
indices: MCV: 110 fl. (N. 80 to 96 fl). MCH: 40 pg (N. 27 to 33 pg) MCHC: 34 g/dl. (N. 33.4 to 35.5
g/dl). WBC and platelet counts: slightly decreased Which of the following is the most probable
type of anemia seen in this patient?
Aplastic anemia from bone marrow depletion
Hemolytic anemia
Iron deficiency anemia
Macrocytic megaloblastic anemia
Macrocytic non-megaloblastic anemia

12) A 50-year-old man with progressive renal failure over 4 years presents with recent onset of
numbness of his feet and hands. Examination shows decreased sensation to pinprick and vibra-
tion stimull below the knee and in hands, and absence of ankle reflexes. Laboratory investigation
revealed: Hematocrit 35% (N. 35.5 to 44.9%). Sodium 140 mEq/L (N. 136-146 mEq/L). Potassium
5.0 mEq/L (N: 3.5-5.2 mEq/L). Urea nitrogen 98 mg/dL (N. 10-20 mg/dL) Creatinine 8.5mg/dL (N.
0.84 to 1.21 mg/dL) Which of the following is the most appropriate next step in management?
Bicarbonate supplements
Blood gas analysis
Peritoneal Dialysis
Recombinant erythropoletin
Vitamin B12

4
13) A4-month-old girl brought to well-baby clinic due to poor weight gain. There is no history of
vomiting or diarrhea. Her examination revealed only weight 60% of expected with loss of subcu-
taneous fat. Which of the following can explain failure to thrive?
Decreased intake
Renal disease
Tuberculosis
Inborn error of metabolism
Celiac disease

14) A one-year-old boy is brought to the emergency room with severe colicky abdominal pain asso-
ciated with vomiting and bleeding per rectum. PR examination reveals a mass and blood on the
surgeon’s finger on withdrawal. Which of the following is the most likely diagnosis?
Gastroenteritis
Intussusception
Polyp
Pyloric Stenosis
Volvulus

15) A 7-month-old boy was brought to the clinic complaining of sudden high fever (40°C), cough,
cyanosis, intercostal retractions and nasal flaring. He had a 3-day history of fever (37.9°C) and
runny nose. Within 24 hours, the patient deteriorated and developed a pneumatocele and left
sided pneumothorax. Which of the following is the most appropriate first action?
Antipyretics
Blood gases
Chest tube
High resolution CT chest
Intravenous antibiotics

16) who presents with confusion, headache and fever is admitted to the Emergency Department.
Shortly after admission she has a seizure. A MRI scan is performed which shows patchy haemor-
rhagic changes in the temporal lobe. Which of the following is the commenst causative organism
of encephalitis?
Pneumococcus
Meningeococcus
Hemophilus influenza
HSV
CMV

17) A 17-year-old female came to Emergency Room with disturbed consciousness. Her mother said
that she found an empty bottle of drug in here room which was found to be acetaminophen.
Which of the following is the most serious complication expected in this patient?
Acute liver cell failure
Chronic liver cell failure
Hepatic adenoma
Hepatic carcinoma
5
Portal hypertension

18) A 60-year-old woman presents with an ulcer on her shin. The ulcer developed two weeks ago
and is enlarging and getting deeper. There is mild surrounding erythema but the patient does
not feel systemically unwell. She has Type 2 diabetes and has a BMI of 32.8kg/m2. Her HbAlc is
110 mmol/mol.On examination, she is afebrile. The wound is exuding a bluish-green fluid with a
fruity odour. A swab taken from the ulcer grows Gram-negative bacilli Which of the following is
the most effective antibiotic for this case?
Flucloxacillin
Metronidazole
Vancomycin
Ceftazidime
Trimethoprim

19) A pregnant woman P3 + 0, came to Ante Natal Care clinic at 8 weeks of gestation. She is con-
cerned that a nurse notified her that you may have a diabetes during this pregnancy. Which of
the following histories might lead you to suspect the existence of diabetes in a such patient?
spontaneous rupture of the membranes occurred during the second trimester in the previous
pregnancies
jaundice appeared in the last trimester of her second pregnancy
both previous deliveries were by cesarean section due to antepartum hemorrhage
unexplained intrauterine fetal death occurred at 38 weeks’ gestation in her last pregnancy
abruptio placentae occurred in the second pregnancy

20) A 34-year-old man presents to the clinic complaining of fever, night sweats, anorexia, cough and
blood-tinged sputum. The chest x-ray reveals infiltrates in both the lower and upper lung lobes,
with possible cavitation in the apices. The case was diagnosed as tuberculosis, Which of the fol-
lowing is the most appropriate preventive therapy after contact’s screening?
Ethambutol
Fansidar
Isoniazid (NH)
Pyraziramide
Rifampicin

21) A ten-year-old boy came to the outpatient clinic with severe headache over the last week. Head-
ache is unilateral, pulsating, and associated with intense vomiting. Which of the following is the
most appropriate management step?
CSF examination
ENT examination
Echocardiography
MRI brain
X-ray skull

22) A 25-year-old presented with galactorrhea, Pregnancy test was negative. Which of the following
is the best next investigation?
Breast US
6
Mammography
Sonomammography
CT brain
Cytology

23) A 40-year-old woman presents with a history of a slowly growing right-sided lump at the base of
the neck. On examination, there is a thyroid nodule with associated lymphadenopathy. She has a
recent history of adrenalectomy. Which of the following is the most likely diagnosis of this thy-
roid mass?
Graves’ disease
Follicular adenoma
Medullary carcinoma
De Quervain’s thyroiditis
Papillary carcinoma

24) A 6 months old healthy infant presents to the immunization room in a primary health care center
for vaccination. Which of the following vaccines should be given to him at this age, according to
Egyptian vaccination schedule?
BCG vaccine, Salk vaccine, Oral polio vaccine and pentavalent vaccine
MMR, pentovalent vaccine and Oral Polla vaccine
Oral Polio vaccine and Pentovalent vaccine
Pentavalent vaccine, rubella vaccine and Vitamin A capsule
Rubella vaccine, Hemophilus influenza b vaccine, pentavalent vaccine

25) A 23year old female comes to the physician because of Swelling in her vulva, she states that the
swelling started about 3 days ago and has been growing larger since, the swelling is painful.
Examination shows a cystic mass 4 cm diameter in posterior aspect of patient’s left Labia, pelvic
examination is unremarkable which of the following is most likely diagnosis
Infected Bartholin’s cyst
Condyloma lata
Gartner’s cyst
Vulvar Concer
urethral curcle

26) During routine follow-up of a 38-year-oid lady on oral contraceptives for 7 years, a Pap smear
was done. The result showed presence of cells suggestive of high grade squamous intra-epitheli-
al lesion (HSIL). Which of the following is the proper further management?
Repeat cytology after 3-6 months
Colposcope directed biopsy.
Fractional curettage
Cervical conization
Hysterectomy

27) A 45-year-patient complained of acute abdominal pain. There was leucocytosis. BP: 90/60, blood
7
sugar: 55mg/dl, Na: 119mEq/dl and K: 6.3mEq/dl. Which of the following is the definitive treat-
ment of this case?
Hypertonic saline
10% dextrose infusion
Laparotomy
Corticosteroids
Vasopressors

28) A 15-year-old boy was brought to chest clinic with attacks of breathing difficulty with whistling
sounds. These symptoms occurred frequently especially after dust exposure. His younger sister
had the same condition. Clinical examination revealed scattered rhonchi all over the chest. Which
of the following is the most likely drug that can exaggerate his condition?
Calcium channel blockers
Metoclopramide
Non-steroidal anti-inflammatory drugs
Paracetamol
Penicillin

29) A primigravida, 35 weeks’ gestation presents with first episode of painless bleeding yesterday.
O/E: BP 120/70 mmHg, uterus relaxed, and cephalic floating fetal head, FHS regular, Hb 10g/dl.
Next line of management is:
Wait and watch
Induction of labor
Blood transfusion
Cesarean section
non of the above

30) A health full term breast-fed infant was brought to the clinic with jaundice at 36 hours of age.
Total serum bilirubin is 20 mg/dl, direct bilirubin is 0.9 mg/dl and reticulocytic count is 10%. The
baby’s blood group is A negative and mother’s blood group is O negative. Which of the following
is the most likely diagnosis?
ABO incompatibility
Breast milk-associated jaundice
Criggler Najjar Syndrome
Hereditary spherocytosis
Rhesus hemolytic disease

31) A 79-year-old man with metastatic colorectal cancer declines chemotherapy as he does not wish
to suffer the side effects of treatment, even though he accepts the chemotherapy may prolong
his life. Which of the ethical principles is best described in this question?
Autonomy
Doctrine of double effect
Non-maleficence
Informed consent

8
Beneficence

32) A diabetic mother gives birth to a full-term neonate with a birth weight of 5 kgs. On examination,
the baby is active and pink with good neonatal reflexes and mild jitteriness. Heart rate & respi-
ratory rate are within normal. Which of the following biochemical tests is essential during the
initial neonatal examination?
Blood glucose
Blood hemoglobin
Serum calcium
Serum magnesium
Serum sodium

33) An 11-month-old boy is brought to the Emergency Department with 6 episodes of diarrhea and
4 episodes of vomiting daily for the past 2 days. On examination, he has abdominal distention,
dry mouth, sunken eyes, and depressed fontanelle. Which of the following is the most important
initial management step?
Giving antidiarrheal medication
Giving parenteral antiemetic
Inserting IV line to correct dehydration
Inserting nasogastric tube for feeding
Sending home advising the mother for giving excess fluids

34) A 73-year-old man complains of left lower quadrant pain over the last 10 days. The pain is con-
stant and severe in the last 72 hours. The patient has a history of sigmoid diverticular disease
over the past 10 years. His temperature is fluctuating reaching up to 39.4°C, the pulse is 95 bpm
and blood pressure is 125/74mmHg. The patient has rigors. There is localized tenderness in the
left iliac fossa but no generalized guarding or tenderness. Intravenous fluids and antibiotics are
started. An urgent CT scan of the pelvis is done showing 19 x 15 cm well localized collection.
Which of the following is the most appropriate treatment?
Formation of a defunctioning loop colostomy
Colonoscopy and colonic stent insertion
Continued IV antibiotics and close monitoring
CT guided drainage
Sigmoid colectomy and end colostomy (Hartmann’s procedure)

35) A 16-year-old primigravida a term, not in labour has a sudden onset of continuous lower abdomi-
nal pain, rapid pulse no fetal heart tones low BP and a tender uterus Most likely diagnosis
Abruptio placentae
placenta previa
Amniotic fluid embolism
Supine hypotensive Syndrome
normal labour

36) A 35year-old woman with three children and using intrauterine device for contraception. She
presents with irregular menstrual cycles at intervals anywhere from 33 to 90 days. Menstrual
flow is usually heavy, and she has noted increasing hirsutism. The medication MOST SUITABLE
9
for her is:
Clomiphene citrate (Clomid)
Combined oral contraceptives
Antiprostaglandin
progesterones
HCG

37) An 18-month-old boy is brought to the Emergency Department with fever (40°C), headache and
two attacks of convulsions over the past two hours. Examination shows he is irritable and has
neck rigidity. His mother states he had fever 38°C and mild nasal discharge. Which of the follow-
ing is the most appropriate investigation to be done?
Blood picture
Blood culture
CT head
Lumbar puncture
Serum electrolytes

38) An 8-month-old boy is brought to the Emergency Room with jerky movement of all limbs. He
was a full-term baby, with no neonatal problems. He is exclusively breast fed. He is afebrile with
height and weight at 50th percentile. Which of the following is the next investigation to be car-
ried out?
Electromyography
Lumbar puncture
Nerve conduction velocity
Serum calcium
Serum sodium

39) A 20-year-old man presents to his GP with a testicular swelling. His serum lactate dehydroge-
nase (LDH) is raised with a mildly raised human chorionic gonadotrophin (HCG) and a normal
alpha- fetoprotein. Which of the following is the most likely diagnosis?
Teratoma
Squamous cancer
Anaplastic cancer
Adenocarcinoma
Seminoma

40) A 60-year-old heavy smoker man came to the physician complaining of hemoptysis for one
month. CT showed bronchogenic carcinoma with liver metastases. The diagnosis was confirmed
by bronchoscopic biopsy. To which of the following persons should you explain the situation and
prognosis?
Brother
Daughter
Son
Patient himself

10
Wife

41) A 23-year-old patient is considering contraceptive methods and preferred to insert IUCD but is
devoutly religious and will not accept a method that may “cause an abortion.” What is the prima-
ry mechanism by which IUCD prevents pregnancy?
creating chronic endometritis
preventing fertilization
inhibiting ovulation
altering tubal motility
decrease sperm motility.

42) A 3-year-old boy came to the outpatient’s clinic with fever (37.8°C), runny nose, malaise and
vomiting. Throat examination shows hyperemia and congestion. Which of the following is the
most appropriate medication to start with?
Acetyl salicylic acid
Domperidone syrup
Vitamin B12
Oral amoxicillin
Paracetamol

43) A 5-year-old boy has an itching sensation around his anus which is worse at night interrupting
him from sleeping well. Which of the following investigations confirms the diagnosis?
Direct stool smear method
Peri-anal swab culturing
Peri-anal swab examination
Rectal swab
Stool analysis by concentration.

44) A 70-year-old woman is admitted with a right hemiparesis. Examination reveals that she is in
atrial fibrillation. CT confirms an ischaemic stroke and aspirin 300mg is commenced. If the pa-
tient is well and develops no new problems, at what of the following points should warfarin be
started?
After 14 days
Immediately
After 7 days
Following a repeat CT at 28 days to exclude secondary hemorrhage
Following a repeat CT at 14 days to exclude secondary hemorrhage

45) A 40 years old women G5.P3+1 was delivered previously by vaginal delivery with unremarkable
obstetric history. She presented at 32 weeks with labor pains and her the cervix is 2cm dilated.
The fetus is transvers lie. The membranes were ruptured and the amniotic fluid completely de-
rived. What is the best method of delivery for this woman?
External version
Internal version and extraction
Oxytocin induction
11
Caesarean delivery
Abdominal support to effect position change

46) 38 years old multigravida women complains of the painless loss of urine, beginning immediately
after coughing, laughing, lifting, or straining. Immediate cessation of the activity stops the urine
loss. Which of the following is the most likely diagnosis?
Fistula
Stress incontinence
Urge incontinence
Urethral incontinence.
UTI

47) A 10-year-old boy was brought to the emergency department with progressive acute abdominal
pain over the past 48 hours associated with vomiting. His parents reported that lately he was
always tired. He had increased urination and excessive water drinking over the past 3 to 4 weeks
before developing the abdominal pain. On examination, the boy is conscious, irritable and has
rapid deep respiration. Abdominal examination shows no tenderness or guarding. Which of the
following is the most likely provisional diagnosis?
Acute renal colic
Acute viral hepatitis
Diabetic ketoacidosis
Food poisoning
Gastroenteritis

48) A 19-year-old woman has had increasing malaise for the past 5 months. On examination, she has
a systolic mummer best heard at the end of the left border of the sternum with congested jugular
veins, enlarged liver and bilateral edema of both lower limbs. Laboratory investigation: Serum
bilirubin: 1.0 mg/dl (N. 0.2-1.2 mg/dL), Serum albumin: 3.6 gm/dl (N. 3.5 to 5.4 g/dL) and Serum
creatinine is 1.5 mg/dl (N. 0.84 to 1.21) Which of the following is the most likely cause of edema
in this patient?
Cardiac edema
Respiratory edema
Hepatic edema
Nutritional edema
Renal edema

49) In a typical neonatal resuscitation, which of the following is the preferred compression-to-venti-
lation ratio?
3:1
2:1
1:1
4:1
5:1

50) 16. A 32-year-old female came to the clinic complaining of irritability, nervousness, hot intoler-
12
ance, and protrusion of the eye. On examination: BP was 140/90 mmHg, pulse 100/m and en-
larged thyroid gland. Investigations were done, and she was diagnosed as Grave’s disease What
is the hormonal assay in this patient?
High TSH, high T3 &T4
High TSH, low T3 &T4
Low TSH, high T3 &14
Low TSH, low T3 & 14
Normal TSH, low T3& T4

51) A 2-month-old infant is brought to the clinic with yellowish discoloration of the skin. The mother
states that stool is pale, and urine is dark. Examination shows enlarged right lobe of liver. Lab
results show elevated AST, ALT, total and conjugated bilirubin. Ultrasound fails to demonstrate
the gall bladder. Which of the following is the most likely diagnosis?
Alpha 1-anti trypsin deficiency
Biliary atresia
Cystic fibrosis
Hepatitis B
Hepatitis A

52) A 27-year-old woman, gravida 3, para 2, delivers twins via an uncomplicated vaginal delivery.
Both placentas are delivered shortly afterward. The patient received regular prenatal care and
experienced no issues during her pregnancy. Over the next hour, she continues to experience
vaginal bleeding, with an estimated blood loss of 1150 mL Vital signs are within normal limits.
Physical exam shows an enlarged, soft uterus. Which of the following is the most appropriate
next step in management?
Methylergometrine
Hysterectomy
Uterine artery embolization
Misoprostol
Bimanual uterine massage

53) A 28 years pregnant woman at 22 weeks gestation was discovered to have asymptomatic bacter-
uria. Which of the following is the suitable action to be given?
Expectantmanagement.
Induction of labor.
Antibiotics.
Diuretics.
Intravenous hydration

54) A 19-year-old PG is expecting her hirst child. she is 12 weeks pregnant by dates she has Vaginal
bleeding and en enlarged for dates uterus in addition no fetal heart sounds are heard the ultra-
sound shows no fetus and no placenta but shout snow storm appearance After diagnosis and
initial management of the woman. the follow up Should Include
weakly HCG
Hysterectomy
13
Radio therapy
Single agent chemotherapy
D&C after one week

55) A 43-year-old woman came to the General Surgical Outpatient clinic complaining of a lump in
her right groin which has been present for 1 year. She has a past medical history of asthma and
is a heavy smoker. On physical examination, a 3 x 4 cm blue non-tender, non-expansile mass is
palpable medial to the femoral pulse. It disappears on lying flat and there is an associated weak
cough impulse when the patient is standing. The presence of dilated tortuous veins in the long
saphenous vein distribution is also noted. Which of the following is the most likely diagnosis of
this swelling?
Femoral aneurysm
Saphenovarix
Lipoma
Femoral hernia
Inguinal lymph node

56) A thirty old man complains of some general weakness. A CBC was done and thrombocytopenia
was evident. Which of the following is the next best investigation after detecting thrombocyto-
penia?
D-dimer
PT
PTT
Reticulocytes
Schistocytes

57) A 75-year-old woman comes to the office complaining of a 2-hour history of sudden-onset se-
vere abdominal pain and per rectum bleeding. Other than a stroke 2 years ago, she keeps well.
She has been on aspirin and ramipril since her stroke and takes digoxin for an irregular heart
rhythm. On examination, she is very distressed; her abdomen is soft with no evidence of peri-
tonism. Bowel sounds are audible. An arterial blood gas shows metabolic acidosis. Which of the
following is the
most likely diagnosis?
Ruptured aortic aneurysm
Acute small bowel ischaemia
Perforated peptic ulcer
Diverticulitis
Irritable bowel syndrome

58) A 30-year-old man presents with a left-sided dragging pain and swelling in the left hemi-scro-
tum. On examination, he has a left-sided swelling that is palpated separately from the testes and
feels like a bag of worms. The swelling disappears on lying supine. Which of the following is the
most likely diagnosis?
Hydrocoele
Indirect inguinal hernia

14
Testicular tumour
Epididymal cyst
Varicocele

59) A 65-year-old woman complained of acute weakness of the left upper and lower limbs since 6
hours. Patient is smoker, with diabetes and hypertension. On examination, patient was semicon-
scious, BP was 150/95 mm Hg. There were left upper motor neuron facial paralyis and flaccid
paralysis of left upper and lower limbs with left extensor planter. Which of the following is the
appropriate initial management?
Blood pressure lowering to normal as soon as possible
Cerebral artery thrombolysis
Heparin administration as soon as possible
Patient hospitalization and admission to ICU
Urgent diabetes control

60) A 20-year-old man with a history of type 1 diabetes mellitus admitted to the ICU unit with wors-
ening confusion, air hunger and dry woody mouth. He was on insulin therapy, but he neglected
therapy for the last 2 weeks after having a fever. Patient is on aspirin, beta blocker, and ACE
inhibitors. On examination: patient is semicomatosed, severely dehydrated, urine output is about
200 ml over 24 hours, HR: regular at 110 bpm, and blood pressure: 80/50mm Hg. Laboratory
data: • Random blood sugar: 450 mg/dL, Na: 140 mEq/L (N. 136-146 mEq/L). K: 5.4 mEq/L (N:
3.5-5.2 mEq/L), Cl: 104 mEq/L (N. 96 and 106 MEq/L) CO2: 19 mEq/L (N. 22 to 29 mEq/L). BUN:
68 mg/dL (N. 10-20 mg/dL) Cr: 2.8 mg/dL (N. 0.84 to 1.21 mg/dL) which of the following is the
most likely diagnosis of this case?
Acute kidney injury
Chronic kidney injury
Hypoglycemia
Nephrotic syndrome
Obstructive uropathy

61) An Obese 63-years old women present with a 3-month history of continuous scanty bleeding.
After adequate history and physical examination what is your recommendation?
Cervical cone biopsy
D&C
Cycling with progestin
Laparoscopy
Official visit every 6month for the evaluation.

62) A 28-year-old unmarried female consulted her doctor about her menstrual pattern during the
last four months. Inspite of having regular menses, she has mid-cycle vaginal spotting and
cramping pelvic pain. Her examination was unremarkable and she has normal pelvic ultrasound
scan as well as a normal vaginal cytology. Which of the the following is the most likely cause of
her complaint?
Ovulatory mid-cycle pain and bleeding.
Anovulatory mid-cycle bleeding.
High estrogen breakthrough bleeding.
15
Undiagnosed bleeding tendency.
Severe mixed vulvo-vaginitis.

63) A 75-year-old patient is found to be acutely short of breath after receiving her third unit of
packed red cells. On examination, there are fine end inspiratory crackles up to mid zones in both
lung fields. Which of the following is the most likely diagnosis?
air embolus
delayed haemolytic transfusion reaction
hypocalcaemia
viral infection
circulatory overload

64) An 18-year-old man came to the Neurology Department complaining of high grade fever, head-
ache, vomiting, and photophobia. He had la ong term history of suppurative otitis media. On
examination, chills, severe prostration and neck stiffness were prominent. The neurologist start-
ed immediate treatment after receiving a CSF sample. CSF profile showed elevated protein and
decreased sugar. Which of the following is the causative organism of this condition?
Bacterial
Fungal
Protozoal
Mycobacteria
Viral

65) A 32-year-old Para 2+1 had her 20 weeks anomaly scan. A placenta previa extending over the
internal cervical os has been identified. What is the appropriate management?
Another trans- abdominal scan at 32 weeks.
To be re-evaluated at time of term delivery.
Re-assessment at 38 weeks.
Trans-vaginal cervical cerclage.
Placental site identification every two weeks

66) A 25-year-old woman PO,GI at 8 weeks’ gestation Came to outpatient clinic with vaginal spot-
ting. ultrasound gestational sac with positive fetal heart has been proven. Which of the following
is the effective therapy for her?
assurance and regular follow up
Daily US monitoring
Increased oral fluid intake
oral progeterone
rectal non steroid antiinflamatory

67) A 19-year-old man comes to the ENT outpatient clinic with a gradually increasing midline neck
swelling. On examination, the swelling is 1 x 1 cm, non-tender and immobile. It moves on protru-
sion of the tongue. Which of the following is the most likely diagnosis?
Goitre
Thyroglossal cyst
16
Pleomorphic adenoma
Cervical rib
Pharyngeal pouch

68) A 9-month-old infant is brought to the GP with delayed sitting and standing. Examination reveals
frontal bossing, anterior fontanel 3 fingers broad, bow legs and pigeon shaped chest. Which of
the following is the most likely elevated laboratory test?
Alkaline phosphatase
Calcuim
Magnesuim
Phosphorus
Vitamin D

69) A 3-year-old boy presents with anorexia, malaise, fever (37.7°C), and maculopapular rash that
started on the face and spread to the body for the past 3 days. On examination, there is retro
auricular and posterior occipital lymphadenopathy. CBC results show: WBC is 4500/ml and direct
platelet count is 11000/mm. Which of the following is the most likely diagnosis?
Kawasaki disease
Measles
Roseola
Rubella
Scarlet fever

70) A 74-year-old woman complained of progressive constipation over the last 9 month. She started
to take oral laxatives which used to help but in the last few weeks she had to defecate every few
days by glycerin suppository. She noticed some abdominal distension and occasionally passed
fluidy stools and mucus. She reported some colic and bowel sounds, both diminished if stools or
flatus could be passed. Which of the following is the most likely diagnosis?
Acute on chronic anal fissure
Irritable bowel syndrome
Habitual constipation
Psudo-obstruction (Ogilviesyndrome)
Colorectal malignancy

71) A 37-year-old woman, p0+2, presents with pregnancy-induced hypertension with blood pres-
sure of 150/100 mmHg at 32 weeks of gestation with no other complications. Subsequently, her
blood pressure is controlled on treatment. If there are no complications, at which gestational age
the pregnancy should be best terminated ?
40 completed weeks
37 completed weeks
36 completed weeks
Await spontaneous onset of labor
non of the above

72) A 44-year-old man with Child’s grade C cirrhosis presented with hematemesis. Upper endoscopy
17
revealed esophageal varices. Which one of the following drugs would be the most appropriate as
immediate treatment?
Isosorbide dinitrate
Omeprazole
Propranolol
Somatostatin
Tranexamic acid

73) A 70-year-old ex-smoker comes to the GP complaining of a 3-month history of weight loss and
worsening dysphagia. He is now unable to swallow soft foods and liquids. He is also experienc-
ing nausea and vomiting on several occasions. Which of the following is the most likely cause of
vomiting?
Gastric carcinoma
Oesophageal carcinoma
Opiate analgesia
Pancreatitis
Gastritis

74) A two-year-old boy is brought by his mother to the Emergency Room with difficult breathing and
coughing for the last 2 days. Temperature is 37 ° C, Respiratory Rate is 34 /min and HR is 100/
min. Examination reveals diminished air entry and wheezes on the right side of the chest only.
Chest X-ray reveals overdistended right lung, especially in expiratory film. Which of the following
is the most appropriate next management step?
Administer steroids
Perform chest physiotherapy
Perform bronchoscopy
Use a nebulizer with ipratropium
Use a nebulizer with beta 2 agonist

75) A 7-year-old boy came to the Nephrology clinic with generalized edema that started as morn-
ing eye puffiness. Blood pressure is 110/70. Urine analysis shows protein 4+. Serum albumin is
2.9gm/dl (3.5-5.5 g/dl) and serum cholesterol is 550 mg/dl (150-199 mg/dl) Which of the fol-
lowing is the most appropriate initial line of therapy?
Furosemide therapy
IV albumin transfusion
IV methylprednisolone
Lipid lowering agents
Oral steroids

76) A 76-year-old woman presented by fracture neck of left femur and treated by total hip replace-
ment. Four days after surgery, the patient developed dyspnea and chest pain. On examination,
the temperature was 37.8, RR was 32/min, and pulse was 105B/min. There was right lower
limb oedema. Chest X-ray showed unremarkable findings. D-dimer was 1012µg/L (N:<250µg/L).
Which of the following is the most likely diagnosis?
Wound infection and sepsis

18
Fat embolism after surgery
Pneumonia complicating anesthesia
Pulmonary embolism
Heart failure due to undiagnosed cardiac valve disease.

77) A 26-year-old primigravida at term presents to labor and delivery reporting irregular contrac-
tions and rupture of membranes 21 hours prior to arrival. She has not received prenatal care but
reports that her pregnancy was uncomplicated. She is afebrile, and electronic fetal monitoring is
reactive with occasional mild variable decelerations. What is the most sensitive test to confirm
rupture of membranes?
White blood count.
C reactive protein
pelvic examination
ferning
Coombs’ test

78) A 45-year-old female complains of lower abdominal pain and vaginal discharge. On examination,
there is cervicitis along with a mucopurulent cervical discharge. The Gram smear of the discharge
shows presence of abundant pus cells, but no bacteria. The best approach to isolate the possible
causative agent would be
Culture on chocolate agar supplemented with hemin
Culture on McCoy cells
Culture on a bilayer human blood agar
Culture on vero cell lines
Non of the above

79) A 45-year-old man was infected by Helicobacter pylori and treated for 2 weeks by the triple ther-
apy. He is asymptomatic now. Which of the following is the best follow up?
Urea breath test
Endoscopy
Barium meal
Histological examination
Culture

80) A 56-year-old man presents to the emergency department complaining of a change in skin color,
fatigue, discomfort in his hand joints, and abdominal pain. He has also noticed increased urine
output and thirst. On examination, his skin appears bronze in color, his liver span is 16 cm, and
there is loss of body hair, and testicular atrophy. His ferritin is 600 ng/mL (15-200 ng/mL), aspar-
tate amino transferase (AST) 130 U/L (8-20 U/L), alanine amino transferase (ALT) 150 U/L (8-20
U/L), and total bilirubin 0.5 mg/dL (0.1-1 mg/dL). Coagulation tests and albumin level are nor-
mal, but the random glucose is elevated at 250 mg/dL. Which of the following is the most likely
provisional diagnosis?
Diabetes mellitus (DM)
Amyloidosis
Wilson disease
19
Hemochromatosis
Addison disease

81) An 80-year-old man whilst micturating suddenly feels sweaty, nauseous and lightheaded. He
describes his vision as though looking into a tunnel before passing out. His past medical history
includes benign prostatic hypertrophy. Clinical examination is normal. What is the most likely
diagnosis?
Subarachnoid haemorrhage
Vasovagal syncope
Hypoglycaemia
Subdural haemorrhage
Hypovolaemia

82) A 28-year-old girl presents with oligo-hypomenorrhea, and facial hirsutism. Her physical exam-
ination is normal, and transvaginal ultrasound show multiple subcortical small cysts 4-6 mm in
both ovaries. Which of the following is the most likely laboratory finding?
Increased FSH level.
Increase in androgens level.
Decrease in estrogen level.
Increased prolactin.
Decreased fasting serum insulin.

83) A 28-year-old man has had a tender red swelling developing at his anal margin over 2 days. He
has had recurrent episodes of similar symptoms with occasional discharge from the area. Exam-
ination reveals temperature 38.1°C, external opening of a fistula in ano and an associated area
of subcutaneous fluctuance. Which of the following is the single most appropriate management
plan?
Examination under general anaesthetic
Immediate antibiotics and review in outpatients
Incision and drainage on the ward
MRI scan
Needle aspiration on the ward

84) A 33-year-old patient has incapacitating menorrhagia. Cyclic oral contraceptive pills previous-
ly had been unsuccessful. She wishes to retain her uterus in hope of becoming pregnant in the
future. Which of the following current treatment options may be helpful for menorrhagia but
should be avoided because of reducing her chances of successful future pregnancy?
tranexamic acid during each period
continuous oral contraceptive pills
levonorgestrel-releasing intrauterine system (Mirena)
endometrial ablation
monthly injection of a long acting gonadotrophin agonist e.g Zoladex

85) A 45-year-old man complains of episodes of palpitations and headache. He has no known histo-
ry of hypertension or other medical problems. He is not a cigarette smoker. On examination: BP:
20
170/100 mm Hg, heart rate: 100 beat/min otherwise, the exam is normal. Laboratory investiga-
tion revealed:• Fasting Blood Sugar: 109 mg/dl (N. 100 to 125 mg/dL) . S. creatinine is 1.3 mg/dL
(N. 0.84 to 1.21) Which of the following is the most likely diagnosis?
Grade 1 hypertension
Grade 2 hypertension
Grade 3 hypertension
Prehypertension state
Systolic hypertension

86) A 75-year-old man presents to the Emergency Department with sudden onset frank haematuria.
Five days earlier he required catheterization following an episode of postoperative urinary reten-
tion after an elective inguinal hernia repair. The catheter was removed on discharge and he has
passed urine freely since. On examination, the urine is frank haematuria with evidence of blood
clots. Which of the following is the most likely diagnosis?
Benign prostatic hypertrophy
Urethral trauma
Urinary tract infection
Transitional cell carcinoma
Renal calculi

87) A nine-month-old infant was brought to the Emergency Department with repeated attacks of
watery diarrhea for the past 24 hours. On examination, he has slow skin pinch, mildly sunken
eyes, and irritability. Which of the following is the most appropriate management step?
IV correction of dehydration
Starting oral rehydration therapy
Ryle tube administration of oral rehydration solution
Stopping diarrhea
Ordering electrolytes and kidney functions

88) A 37-year-old, Para 1+2, came for antenatal care at 10 weeks gestation. Her first 4.5 kg baby
was delivered vaginally after 2 abortions. She has no history of gestational diabetes but a posi-
tive family history. Her body mass index is 27 kg/m2. What puts her more under a risk to develop
gestational diabetes?
Her previous macrosomic baby.
Her previous two abortions.
Her body mass index
Her family history for diabetes.
Her age in the current pregnancy.

89) A 60-year-old female Came to the outpatient Clinic with Vaginal bleeding that started 10 days
ago. Her menopause was 6 years ago, and her gynecological history was unremarkable before
that. Which of the following is the primary management for her?
Hysterectomy
Cervical core biopsy
Endometrial biopsy
21
Laparoscopy
MEI

90) A 59-year-old banker with no past medical history presents to the emergency department 4
hours after the onset of severe substernal crushing chest pain with radiation to the left arm
and neck. Electrocardiography reveals significant ST-segment elevation in leads I, L, V5, and V6.
Which of the following is the most likely diagnosis?
Anterior infarction
Inferior infarction
Lateral infarction
Posterior infarction
Unstable infarction

91) A 52-year-old woman comes to the office because of a 2-day history of pain in her right calf
when walking that occurs in the evening. Twelve hours later, she had onset of rapid swelling
of her right leg and she now has difficulty in walking. She says her symptoms began after she
returned from a long vacation flying for 10 hours. She has a 10-year history of type 2 diabetes
mellitus treated with metformin. Her temperature is 37°C, pulse is 75/min, respirations are 16/
min, and blood pressure is 120/70 mm Hg. Physical examination shows edema and tenderness
(pain) to palpation of the right calf. No erythema is noted. Pedal pulses are normal. Which of the
following is the most likely diagnosis?
Arterial embolism
Cellulitis
Deep venous thrombosis
Diabetic foot infection
Myositis

92) A two-and-a-half-year-old boy is brought to the pediatrician for his annual physical examination.
His mother is worried as he has poor interaction with peers and family, delayed language de-
velopment, repetitive movements and does not make eye contact. Which of the following is the
most likely diagnosis?
Attention deficit disorder
Autism spectrum disorder
Childhood disintegrative disorder
Anxiety disorder
Intellectual disorder

93) A 65-year-old man complains of progressive abdominal distension, colicky pains and reduced
bowel frequency for 8 days. He has vomiting for the last 12 hours. Which of the following is the
most appropriate first investigation?
Barium enema (double contrast)
Colonoscopy
CT scan of the abdomen and pelvis
Endoanal ultrasound scan
Flexible sigmoidoscopy
22
94) A7-year-old boy brought to the emergency room with generalized urticarial rash 15-minutes fol-
lowing ingestion of nuts. Which of the following is a clinical manifestation of anaphylactic shock?
Cough
Hypotension
Hypertension
Cold skin
Sore throat

95) A 26-year-old man is brought to the emergency department with sudden onset breathlessness.
Examination shows him to be dyspneic at rest, with saturations of 80% on room air and a pulse
rate of 100 beats/min. His trachea is not deviated, and his blood pressure is 117/82 mm Hg. X
ray revealed a pneumothorax. After administering oxygen, which of the following is the next step
of management?
Intravenous furosemide
Intravenous hydrocortisone
Nebulized salbutamol
Non-invasive ventilation
Intercostal tube insertion

96) A 2-year-old girl comes to the clinic complaining of fever, bruising and generalized petechiae all
over her body. Temperature is 37.8 °C. Two weeks before, she had rhinitis and mild cough for 3
days. Examination shows no other physical abnormalities. Hemoglobin is 12.3 gm/dl and platelet
count is 21,000/mm3 Which of the following is the most likely diagnosis?
Acute leukemia
Aplastic anemia
Immune thrombocytopenic purpura
Systemic lupus erythematosus
von Willebrand disease

97) A 40-year-old woman came to the hospital for follow up of chronic liver disease. She said that
she was diagnosed as primary biliary cirrhosis Which of the following is the etiology of liver cir-
rhosis in this patient?
Autoimmunity
Bacterial infection
Blood disease
Metabolic cause
Viral infection

98) A one-year old boy is brought to the clinic by his mother for routine well child examination. He
is exclusively breast-fed till now. The mother says she tried to introduce cereals, but he refused.
On examination, the child is pale with no organomegaly. The GP requests a blood picture which
shows a Hg 8.7 gm/dl (N: 14-17 gm/dl) and HCT 24%. Which of the following is the most likely
diagnosis?
Thalassemia
Iron deficiency anemia
23
Pernicious anemia
Sideroblastic anemia
Spherocytosis

99) A 65-year-old man has an enterocutaneous fistula originating in the jejunum secondary to in-
flammatory bowel disease. Which of the following would be the most appropriate fluid for re-
placement of his enteric losses?
D5W. Dextrose 5%
3% normal saline
Ringer lactate solution
0.9% sodium chloride
6% sodium bicarbonate solution

100) A female patient, 35 years old presented to the emergency room with frequent loose stool and
passage of small amount of red blood per rectum. She recalled that she has frequent attacks
of diarrhea and central abdominal pain over the last 5 months. Colonoscopy revealed scattered
patchy areas of ulcerations alternated with healthy areas. What is the most likely cause of the
bleeding in this?
Cancer colon
Crohn’s disease
Esophageal varices
Ischemic colitis
ulcerative colitis

18 November 2023 (1st trial exam)

1) A 56-year-old man with chronic renal failure missed his hemodialysis yesterday. He presents with
his wife complaining of weakness and lethargy. His serum potassium is 7.6 mEq/L Which of the
following agents acts by shifting the potassium from the extracellular space to the intracellular
space?
Calcium gluconate
Furosemide
Hemodialysis
Insulin
Sodium polystyrene sulfonate

2) A 35-year-old woman came to ER complaining of palpitation, cold extremities and dizziness. By


examination, blood pressure was 60/40 and HR was 200 b/min. ECG showed wide QRS complex
tachycardia. Which of the following is the most appropriate first approach?
Intravenous beta-blockers
Intravenous calcium channel blockers
Termination of tachycardia by DC cardioversion
Intravenous amiodarone
Carotid sinus attack

24
3) A 50-year-old man presented with mass in the right lobe of the liver. He is chronic alcoholic with
known cirrhosis. There is an elevated a-fetoprotein level. Which of the following is the most like-
ly diagnosis?
Hepatocellular adenoma
Hepatocellular carcinoma
Metastatic carcinoma of the colon
Regenerating nodule of cirrhosis
Focal nodular hyperplasia

4) A newborn female has been diagnosed as having congenital diaphragmatic hernia immediately
after birth. Which of the following is the first intervention to be done?
Inserting a nasogastric tube
Inserting a central venous line
Bag and mask ventillation
Oral feeding with syringe
Inserting a rectal tube

5) A 3-month-old previously healthy infant is brought to the emergency room with a generalized sei-
zure. The physical examination shows a temperature of 36.5°C, respiration rate of 50/min, and
marked hepatomegaly, with the liver edge palpable down to the umbilicus. Laboratory findings
show Na, 141 mEq/L; CI, 95 mEq/L; K, 4.5 mEq/L; BUN, 12 mg/dl; glucose, 25 mg/dL; uric acid,
7.1 mg/dL; cholesterol, 270 mg/dl; and triglyceride, 550 mg/dL Which of the following is the
most likely
diagnosis?
Hyperinsulinemia
Adrenal insufficiency
Growth hormone deficiency
Glycogen storage disease
Mitochondrial disorder

6) Which of the following is the cause of raised phosphate level in patients with chronic kidney dis-
ease?
Decreased renal excretion
Increased gut absorption
Hypervitaminosis D
Primary hyperparathyroidism
Decreased 2d alpha hydroxylation of vitamin D

7) A 48-year-old man has intermittent left sided lower abdominal pain and progressive constipation
and feels generally unwell. He has lost his appetite and lost weight. His temperature is 37.3°C,
his blood pressure is 150/100 mmHg, and his pulse is 80 bpm. What is the best investigation
which lead to diagnosis?
Colonoscopy
Fasting serum glucose level
MRI abdomen
25
Trans rectal ultrasound
Ultrasound abdomen

8) A 6-year-old girl came to the office complaining of morning sneezing, clear rhinorrhea, and poor
sleep. Her body temperature was 37° C and she had dark halos below the eyes. Which of the fol-
lowing is the most likely diagnosis?
Foreign body
Vasomotor rhinitis
Chronic sinusitis
Allergic rhinitis
Influenza infection

9) A 75-year-old man has been resuscitated for more than 45 minutes following a cardiac arrest, and
after excluding all reversible causes. There is no sign of return of the ROSC (=Return Of Sponta-
neous Circulation) and a decision has been made to stop resuscitation. His relatives are waiting
and are unaware of the decision. Which of the following is the most appropriate step to be taken?
Bringing the relatives into the resuscitation room to break the news to them
Sending a junior doctor to break the news
Sending a nurse to reassure them that the team is still trying its best
Sending a senior nurse with security man to break the news in the waiting room
Meeting of the team leader with the relatives in the relative’s room to break the news

10) A 35-year-old firefighter is admitted from a house fire with 50% third-degree burns. His weight is
estimated at 80 kg. Which of the following is the amount of fluid that should be administered to
him in the first 8 hours?
5L
6L
8L
14 L
16 L

11) A 10-month-old girl was brought to the office complaining of abdominal pain and diarrhea,
which has recently had pink staining. She was well until 12 hours ago, when she had some loose
stools and episodes of crying and holding her abdomen. On examination, she looks intermittently
uncomfortable. Her abdomen is slightly distended and she has a curved firm mass 10 cm long ex-
tending from the right iliac fossa towards the hepatic flexure. Which of the following is the most
likely diagnosis?
Inborn errors of metabolism
Constipation/encopresis
Intussusception
Renal anomalies
Malignancy e.g. neuroblastoma, Wilm’s, lymphoma

12) Which of the following is a cause of microcytic anemia?


Thalassemia
Sickle cell anemia
26
Vitamin B12 deficiency anemia
G6PD deficiency
Autoimmune hemolytic anemia

13) A 79-year-old woman complains of becoming progressively forgetful in the past 9 months. She
has trouble remembering familiar places and people and can no longer balance her checkbook,
and has increased trouble to verbally expressing her thoughts. These symptoms have progres-
sively worsened in the past several months. She has no history of head trauma or of anxiety
disorder. Her neurologic examination is within normal limits except for a mini-mental status
examination score of 22 of 30, missing points on calculation, recall, and orientation. A rapid plas-
ma reagin antibody test (to assess allergic reactions) was negative, and the thyroid-stimulating
hormone, vitamin B12, folate, and electrolytes were normal. Which of the following is the most
likely diagnosis?
Alzheimer’s disease
Creutzfeldt-Jakob disease
Delirium
Depression
Normal pressure hydrocephalus

14) A 3-year-old boy just says mama and dada with no eye contact. Mother says he is not olaying
with others at the nursery but frequently runs in circles. Which of the following is the most likely
diagnosis?
Autism spectrum disorder
Anexiety disorder
Deaf mute
Attention deficit hyperactivity
Cerebral palsy

15) A 28-year-old, Para2 used combined oral contraceptive pills for the last 8 months as advised by
one of her friends. Her mother died of breast cancer 3 years ago. What is the appropriate advice
regarding contraception?
Request regular serum sex-hormone profile.
Regular screening for breast cancer.
Shift to another non-hormonal method.
Ask for pathology report of her mother’s tumor.
Use prophylactic anti-estrogen drugs

16) An 18-year-old boy is presents with bloody diarrhea over the past 2 weeks, accompanied by
frequent urges & abdominal cramping. One hour ago, he saw mucus & fresh blood on his stool.
Similar symptoms have occurred over the past 2 years, except for the blood in his stool. His
temperature is 37.5°C, BP 120/70 mm Hg, pulse 65/bpm. His abdomen is soft, without guarding.
Which of the following
is the most likely diagnosis?
Appendicitis
Diverticulitis

27
Ischemic colitis
Pseudomembranous colitis
Ulcerative colitis

17) A 5-year-old child has a postive urine dip for nitrites and leucocytes. Which of the following sug-
gests that pyelonephritis is present?
Dysuria
Enuresis
Frequency
Temp > 38 degrees
Mild abdominal pain

18) A 32-year-old, Para4 committed an illegal abortion at 8 weeks gestation by an unqualified doctor
at his clinic. One week later she presented with moderate vaginal bleeding with offensive odor
her temperature39 °c. Pulse 120/m, blood pressure 100/60. Which of the following is the most
likely diagnosis?
Incomplete evacuation of conceptus
Incomplete septic abortion
Undiagnosed extra-uterine pregnancy.
Trichomonas vaginalis infection
Severe urinary tract infection.

19) A previously healthy, obese 26-year-old G1 PO has a positive glucose challenge test at 28 weeks
gestational age (GA). Follow-up testing confirms the diagnosis of gestational diabetes mellitus
(GDM). The patient asks what this diagnosis will mean for her baby. Which of the following can
be a significant consequence of poorly controlled GDM to the fetus?
Hyperglycemia
Macrosomia
Cataracts
Cerebral edema
Cephalohematoma

20) A 45-year-old woman with ulcerative colitis is admitted with a history of jaundice, pruritus, and
intermittent abdominal pain. Examination shows hepatosplenomegaly and mild ascites. Blood
tests confirm an obstructive jaundice, and mitochondrial antibodies are not detected. Which of
the following is the most likely diagnosis?
Chronic active hepatitis
Liver cirrhosis
Metastatic carcinoma
Pancreatic carcinoma
Sclerosing cholangitis

21) A 3.8 kg full term boy was delivered by elective CS. Apgar score was 5 & 7 at 1 & 5 minutes re-
spectively. Pregnancy and delivery were unremarkable. Temperature is 36.50 C, Respiratory Rate
is 70 per minute, Heart Rate 140 per minute, and oxygen saturation SpO2 92% on nasal oxygen
28
2 L/min. Chest examination revealed moderate chest retractions, grunting, and fair air entry.
Examination of heart, abdomen, and CNS were unremarkable. Which of the following is the most
likely diagnosis?
Congenital pneumonia
Meconium aspiration
Respiratory distress syndrome
Tracheoesophageal fistula
Transient tachypnea of the newborn

22) A 32-year-old Para 2, presented to the emergency department shocked with lower abdominal
pain and mild vaginal bleeding. Her pregnancy test was positive and ultrasound showed an emp-
ty uterus and hemoperitoneum. She was diagnosed as acutely disturbed ectopic pregnancy. She
underwent a laparotomy and left salpingectomy followed by cervical dilatation and endometrial
curettage. Which one of the following pathologies of her curettage would be most likely?
Chorionic tissue.
Arias Stella reaction.
Simple endometrial hyperplasia.
Complex hyperplasia.
Atrophic endometrium.

23) An 18-year-old male with known sickle cell is admitted to the haematology department with
tiredness and yellowing of his eyes following a recent change in weather. You note his haemoglo-
bin usually runs between 8-9mg/dL but an up to date blood test shows his Hb is now 5.3mg/dL.
You suspect he is suffering from a haemolytic crisis. Normal HB level (12-16 mg/dl). Which of the
following types of bilirubin is/are increased in intravascular hemolysis?
Conjugated bilirubin
Unconjugated bilirubin
Both conjugated and unconjugated bilirubin
Haptoglobin
Lactate dehydrogenase

24) A normally fit and healthy 68-year-old man presented to the Emergency Department after a sud-
den collapse in the street with sudden onset back pain. On examination he was hypotensive and
a palpable pulsatile expansile epigastric mass was found, and a diagnosis of ruptured abdominal
aortic aneurysm was made.. Which of the following is the appropriate management of abdominal
aortic aneurysm?
Do nothing
High flow oxygen
Open surgical repair
IV opioid analgesia
IV fluids

25) A 32-year-old Para 2, both deliveries were by CS. Her routine ANC is unremarkable, Ultrasound
done at 32 weeks shows that the placenta is grade 2 and located anteriorly down to the level of
internal os. Which one of the following conditions is the most likely risk?
Placenta accreta.
29
Abruption placenta.
Cerebrovascular accident.
Amniotic fluid embolus
Pulmonary edema.

26) A 3-year-old boy was brought to the Emergency Room with a prolonged generalized seizure.
Venous access could not be established. Which of the following drugs is recommend to be given
through rectal administration?
Diazepam
Midazolam
Phenobarbital
Phenytoin
Valproic acid

27) A 55-year-old man presents to the Emergency Department with a 2-day history of vomiting and
severe epigastric pain radiating through to his back. He admits to a large alcohol binge the night
before the onset of the pain. On examination, he is tachycardic with generalized abdominal ten-
derness and localised guarding in the epigastrium. An erect chest X-ray shows no obvious abnor-
mality. You suspect a diagnosis of acute pancreatitis. The surgical registrar asks you to ‘score’
the patient. Which of the following forms part of the Glasgow Coma Score for acute pancreatitis?
Heart rate
Systolic blood pressure
Temperature
PaO2
Amylase

28) A 50-year-old woman complained of bleeding per rectum. There is a soft anal swelling that pro-
trudes with defecation and the patient can reduce it manually. Which of the following is the most
likely provisional diagnosis?
Ist degree piles
2nd degree piles
3rd degree piles
4th degree piles
Anal fissure

29) A 31-year-old woman presents with increased vaginal discharge and dysuria of two days du-
ration. She is sexually active with her husband and uses condoms intermittently. Examination
reveals some erythema of the cervix. Urine culture is negative. Sexually transmitted disease
testing revealed gonorrheal infection. Which one of the following infections should be treated
concomitantly?
Bacterial vaginosis.
Chlamydia
Herpes.
Syphilis.
Trichomoniasis.
30
30) A 63-year-old man presents with a 2- day history of a dull left-sided loin pain. He has a known
history of a left sided renal calculus. On examination, his temperature is 38.5° C and he has
rigors, Urea 9.9 mmol/L and creatinine 155 µmol/L. A CT KUB confirms a large left pelvi-ureteric
junction calculus with left sided hydronephrosis. Which of the following is the appropriate man-
agement?
Extracorporial shockwave lithotripsy (ESWL)
Conservative management
Percutaneous nephrolithotomy (PCNL)
Ureteric stenting
Open surgical extraction

31) Precocious puberty associated with bony dysplasia and café au lait spots on skin is noted in a 6
years old girl. Which of the following is the most likely diagnosed?
Laurence-Moon-Biedl syndrome
McCune-Albright syndrome
Alport’s syndrome
Frohlich’ssyndrome
Syphilis

32) A 15-year-old adolescent girl is diagnosed with meningococcemia. She has a 4-year-old brother
and an 11-month-old sister at home. Which of the following is the appropriate management of
her siblings?
Nasopharyngeal cultures for N.meningitidis
Meningococcal quadrivalent vaccine
Single dose of azithromycin to both children
Rifampicin given every 12 hours for 2 days
Close observation for a febrile illness

33) A 72-year-old woman came to the Emergency Department complaining of acute left lower quad-
rant abdominal pain. Her past medical history is significant only for mild hypertension. On ex-
amination, her temperature is 38.2 °C with focal rebound tenderness in her left lower quadrant.
WBC count is 12000 /mm3. Abdominal CT imaging reveals a thickened sigmoid with a 3.0 cm
x 3.0 cm peri-sigmoid colon abscess with a very small pocket of extraluminal air in the pelvis.
Which of the following is the most likely next step in management?
Exploration, sigmoidectomy with descending end colostomy (Hartmann’s procedure)
Exploration with sigmoidectomy, primary colorectal anastomosis
Exploration with sigmoidectomy, primary colorectal anastomosis, and loop ileostomy
Resuscitation, broad spectrum IV antibiotics and percutaneous drainage of the abscess
Resuscitation, broad spectrum IV antibiotics, colonoscopy for likely colon carcinoma on this ad-
mission

34) A 35-year-old Para 3 has painless vaginal bleeding at 37 weeks’ gestation. The next measure
immediately indicated is:
Cesarean section.
Induction of labor,
31
Coagulation profile.
Rupture of membranes.
Fetal monitoring.

35) A 7-year-old girl is diagnosed with appendicitis after returning to the ED a day after first pre-
senting with abdominal pain. Appendicitis was intitially misdiagnosed as which of the following
cases?
Mesenteric adenitis
Intussusception
Gastroenteritis
Inflammatory bowel disease
Pancreatitis

36) A 3-year-old boy is brought to the emergency room with sudden difficulty of breathing, cough
and cyanosis. The boy was active all through the day and developed difficult breathing while
playing with his lego bricks. Which of the following is the most likely diagnosis?
Bronchiolitis
Bronchitis
Bronchopneumonia
Croup
Foreign body aspiration

37) A 63-year-old man presents with a large hematoma on his thigh. He has an 8-year history of
hepatitis C infection and well-documented cirrhosis and portal hypertension. On preoperative
screening, his prothrombin time is noted to be 17.4 seconds (N: 11 to 13.5 seconds). Which of the
following is the most appropriate transfusion as a next step in management of this patient prior
to his procedure?
Cryoprecipitate
Fresh frozen plasma
Packed red blood cells
Platelets
Whole blood

38) 26-year-old Para 1+2 came to the emergency department 3 weeks after an abortion, with severe
vaginal bleeding. She reported undergoing surgical evacuation 3 weeks earlier, but did not return
to receive the pathology report of the specimen. Her beta-subunit titre was above 100,000 mil-
li- international units/milliliter, her ultrasound showed an intra-uterine mass 4x4 centimeters.
What is the most appropriate management?
Resuscitation followed by conservative management
Resuscitation and uterotonics
Resuscitation and re-evacuation
Start chemotherapy
Hysterectomy

39) A 30-year-old, G5, Para 0+4 was diagnosed to have antiphospholipid syndrome. She is on low
32
molecular weight heparin till 30 weeks gestation. She starts to have mild hypertension, protein-
uria (+) with normal kidney functions but liver enzymes in the high normal range. What is the
proper plan for management?
Immediate termination of pregnancy.
Termination of pregnancy at 34 weeks.
Termination of pregnancy at 36 weeks.
Termination of pregnancy at 38 weeks.
Termination of pregnancy at term.

40) A 30-year-old primigravida presents at 34 weeks’ gestational age with blood pressure of
170/100 mmHg, headache, epigastric pain, visual abnormalities and 3+ proteinuria. Biophysical
profile of the fetus is 8/8. Which one of the following is the immediate response?
Start antihypertensives
Immediate refer for emergent termination
Give betaclomethasone to induce fetal lung maturity.
Perform an amniocentesis to assess fetal lung maturity.
Repeat the biophysical profile daily.

41) 18 year old female at 7 weeks gestation by LMP complains of 2 day history of Vaginal spotting
And lower abdominal pain Physical examination reveals 4 week Sized uterus. On U/S No intra-
uterine gestational sac in noted What is the next step management?
Observation
Do B-hCG level
Give her Kapron to stop bleeding
Ask her to do U/s after 2 days
Give methotrexate

42) A 3-month-old infant is found to have a continuous murmur all over the precordium during her
routine well baby check up. The infant is 6 kg in weigh and appears well. All peripheral pulses are
present and easily palpable. Oxygen saturation is 96%. Which of the following is the most likely
finding
on echocardiography?
Atrial septal defect
Coarctation of the aorta
Dextrocardia with situs inversus
Persistent ductus arteriosus
Ventricular septal defect

43) A woman presented 5 days after delivery, complaining of a painful swollen left leg. She is found
to have leukocytosis and low grade fever. What is the most appropriate immediate management
before diagnosis of DVT is confirmed?
Warfarin oral treatment.
Prophylactic heparin.
Therapeutic heparin.

33
Low dose aspirin tablets.
Diosmin (venotonic) tablets.

44) A 3-month-old baby is brought to the physician complaining of low grade fever, wheezing and dry
cough. On examination: RR 72/min with hypoxia. CXR shows hyperinflation and some infiltrate.
Which of the following is the most likely diagnosis?
Croup
Epiglottitis
Bronchial asthma
Bronchiolitis
Pneumonia

45) P4+0 64 years old female came to outpatient clinic to insert IUD after 2 months She complains of
abnormal uterine bleeding. Which of the following explanation suggested as etiology to her?
IUD in the device
Unbalanced ratio prostaglandin and thromboxane
Decreased endometrial vascularly congestion and degeneration
Endometrial cancer
Perforation of the uterus

46) A 24-year-old woman GO married for 5 months presents with a non-tender cystic mass in her
right vulva that causes some discomfort when walking and during coitus. The mass was at the
posterior part of labium major and was about 2.0 X 2.0 cm dimensions. What is the MOST AP-
PROPRIATE initial decision?
Marsupialization
Administration of antibiotics.
Observation.
Surgical excision
Incision and drainage

47) A 36-year-old woman presents with a 2-month history of recurrent severe right-side headaches
associated with vomiting and blurring of vision. She feels generally unwell, lethargic and nause-
ated about 12-24 hours before each episode. Which of the following is the cause of this head-
ache?
Giant cell arteritis
Subarachinoid hemorrhage
Tension headache
Cluster headache
Migraine

48) A 70-year-old man presents with a right inguinal mass that has enlarged and has caused discom-
fort in recent months. The patient is a cigarette smoker. He complains of recent difficulty with
micturition and nocturia. The swelling does not extend to the scrotum and reduces when resting.
Which of the following is the most likely diagnosis?
Direct inguinal hernia
34
Hydrocele
Aneurysm of the femoral artery
Cyst of the cord
Strangulated indirect inguinal hernia

49) A healthy baby boy is born at term. His mother was confirmed with acute hepatitis B during this
pregnancy.Which of the following preventative interventions should be given to the baby?
Full course of hepatitis B vaccine and immunoglobulin at birth
Full course of hepatitis B vaccine at birth
Hepatitis B immunoglobulin at birth
Hepatitis B immunoglobulin at birth if baby is HBSAG positive
Hepatitis B immunoglobulin at birth if baby is HBSAG positive

50) A 2-year-old child presents to the clinic with fever 38.5°C, generalized erythematous maculo-
papular rash, cough, coryza and red eyes. He was diagnosed by his doctor as measles infection.
Which of the following complications is considered as long-term for measles?
Interstitial pneumonia
Myocarditis
Otitis media
Pneumomediastinitis
Subacute sclerosing panencephalitis

51) A 74-year-old woman presents with a 3-month history of a painful ulcer on the dorsum of her
foot. The ulcer is punched out, has a deep, pale, minimally sloughy base, and is surrounded by
tight, thin dry skin. Which of the following is the most likely aetiology?
Venous ulcer
Arterial ulcer
Neuropathic ulcer
Diabetic ulcer
Malignant ulcer

52) An 18-month-old male baby is brought to the Emergency Department with diarrhea and vomit-
ing for 2 days. He has about 6 episodes of diarrhea and 4 episodes of vomiting per day and is not
feeding well. His mother states that his activity level is decreased, is always weak and tired and
has a decreased number of wet diapers. Which of the following is the most important initial man-
agement step?
Insert IV line to correct dehydration
Insert nasogastric tube for feeding
Give antidiarrheal medication
Give parenteral antiemetic
Send home advising the mother to give excess fluids

53) A 70-year-old man complained of intermittent left iliac fossa pain, anorexia, and anergia since 5
weeks. He has lost 2kg of weight. He has had low-grade fever for the last 3 nights. There is a ten-
35
der mass in the left lower quadrant. Which of the following is most likely this abdominal mass?
Appendix mass
Colonic carcinoma
Lymphoma
Palpable bladder
Pelvic abscess

54) A 72-year-old man complained of pain in his right big toe. The pain started this morning and
is getting progressively worse, and he is unable to bear weight on that foot. He is febrile, has
a pulse of 90/min, and has a blood pressure of 136/86 mm Hg. The first metatarsophalangeal
joint of the right foot is swollen, warm, and erythematous. It is tender to palpation and there is
decreased movement. Which one of the following is the best agent for the acute management of
this patient’s condition?
Intravenous colchicine
Oral acetaminophen
Oral allopurinol
Oral indomethacin
Oral probenecid

55) A frail 89-year-old man has had of a large pressure sore on the sacrum for the past 2 months.
Medical history includes type 2 diabetes mellitus and multi-infarct dementia. He is incontinent of
stool and urine. Which of the following factors is the most important contributor to the develop-
ment of the pressure ulcer in this patient?
Fecal and urinary incontinence
Inadequate cushioning on the wheelchair
Malnutrition
Poorly controlled diabetes mellitus
Prolonged bed rest

56) A 27-year-old woman complained of a 5-year history of abdominal bloating, weight gain and
constipation, as well as recent anal discomfort with no bleeding. The bloating was eased by
bowel opening, which occurred every 5-7 days. Stool consistency varied from pellet-like to semi-
formed. She strained at stool on every occasion. Which of the following is the mostly likely find-
ing on rectal examination?
A tear in the anal skin is normal with chronic straining, and unlikely to be a cause of her pain
Contraction of the internal anal sphincter is normal on bearing down
Low resting anal pressure excludes an anal fissure
Rectal prolapse is possible despite her young age
The presence of stool in the rectum confirms the diagnosis of faecal impaction

57) room with a history of 2 eclamptic fits at home. Examination revealed blood pressure 180/110,
marked edema, heavy proteinuria. No fetal cardiac pulsations could be detected by ultrasound
scan. What is the appropriate management?
Immediate termination of pregnancy.
Stabilization of her condition and then termination after 1-2 hours.
36
Anticonvulsant and antihypertensive drugs for 24hours.
Conserve until spontaneous expulsion of the dead fetus.
No need to control fits or hypertension as the baby is dea

58) A 36-year-old woman complains of increasing weight gain, tiredness and neck swelling. On ex-
amination, the patient has a firm rubbery goitre. Fineneedle aspiration shows a diffuse lympho-
cytic and plasma cell infiltrate. The report also mentions the presence of lymphoid follicles and
parenchymal atrophy. Which of the following is the most likely diagnosis?
follicular carcinoma
Hashimoto’s disease
anaplastic carcinoma
endemic goitre
lymphoma

59) A 30-year-old woman complained of generalized abdominal pain, vomiting, distention and fever.
Blood picture showed leukocytosis and abdominal ultrasound showed pelvic collection. Which of
the following is the most likely diagnosis?
Perforated appendicitis.
Salpengitis.
Complicated Meckel’s diverticulum.
Amoebic colitis.
Ulcerative colitis.

60) A 54-year-old-woman cuts her hand with a kitchen knife. There is evidence of tendon injury.
Which of the following is true regarding the muscles of the hand?
The flexordiagitorum superficial is inserted on the distal phalanx
The flexordiagitorumprofundus inserts on the middle phalanx
The tendons of the flexor diagitorum superficial is arised from a common muscle belly.
Repair should not be doneimmedialety.
The process of healing a tendon injury involves formation of a tenoma.

61) A 21-year-old woman complained of shortness of breath and productive cough for 2 months.
She came to the outpatient clinic because she noticed bilateral leg swelling. Pulse is 105 b/min
and Bp is 100/60. There is raised Jugular venous pressure (JVP) with bilateral pitting lower limb
edema. There is also loud 1st heart sound with mid-diastolic murmur over her cardiac apex and
ejection systolic murmur over the 2nd left intercostal space. Chest X-ray revealed cardiomegaly.
Which of the following is the most likely diagnosis?
Mitral stenosis
Mitral regurgitation
Aortic stenosis
Aortic regurgitation
Ventricular septal defect

62) An 82-year-old woman presents with a 24-hour history of a painful swelling in her left groin. The
patient is unwell and dehydrated. Pulse is 110 beats/min. On examination, the swelling is 3 x
37
3cm in size, firm and tender. It is not reducible. Which of the following is the most likely diagno-
sis?
Inguinal lymphadenopathy
Femoral hernia
Inguinal hernia
Saphena varix
Femoral artery aneurysm

63) A 34-year-old primigravida, at 28 weeks, presented at her routine antenatal visit with a com-
plaint of vaginal discharge diagnosed to be vaginal candidiasis, for which she received two suc-
cessive courses of local treatment with incomplete resolution. Her urine analysis showed pus
cells 50- 60/high power field, sugar 1+. On ultrasound examination, the amniotic fluid index was
23. What is the next step of management?
Schedule for the next prenatal visit after 2 weeks.
Repeat local treatment for infection using another preparation.
Request a glucose tolerance test.
Systemic antifungal treatment
Termination of pregnancy

64) A 5-year-old boy was brought to ER by his mother complaining of drooling saliva, inability to eat
and drink. On examination, there were fever, congested pharynx and inspiratory stridor. Which of
the following is the most likely diagnosis?
Viral pneumonia
Croup
Acute epiglottitis
Bacterial pneumonia
Bronchiolitis

65) A 30-year-old executive learns that he has a duodenal ulcer. His gastroenterologist prescribes
and outlines medical therapy. The patient worries that if medical therapy fails he may need sur-
gery. Which of the following is the best indication for elective surgical therapy of duodenal ulcer
disease?
An episode of melena
Repeated episodes of pain
Pyloric outlet obstruction due to scar formation from an ulcer
Frequent recurrences of ulcer disease
Referral of pain to the back, suggestive of pancreatic penetration

66) A 60-year-old woman comes to the emergency room in a coma. The patient’s temperature is
32.2°C. She is bradycardic with puffy eyes and face. Her thyroid gland is enlarged. There is dif-
fuse hyporeflexia. BP is 100/60. Which of the following is the best next step in management?
Await results of T4 and TSH.
Obtain T4 & TSH; begin IV thyroid hormone & glucocorticoid.
Begin rapid rewarming
Obtain CT scan of the head.
38
Begin intravenous fluid resuscitation.

67) A 25-year-old G3 P1+1 presents to the emergency room complaining of lower abdominal crampy
pain 6 weeks from her last normal period. She had significant vaginal bleeding but no passage of
tissue & pregnancy test is Positive. Which of the following is the next step in this patient’s asses-
ment?
Sonography.
Physical exam
CBC
Quantitative B-hCG.
Detailed menstrual history.

68) A 35-year-old non-lactating Para 2 has inserted an intra-uterine contraceptive device 6 weeks
after her last delivery. Two weeks later, she started to notice offensive vaginal discharge, low-
er abdominal pain and a temperature of 38°c. Her urine culture revealed insignificant bacterial
growth. C-reactive protein was elevated and pregnancy test was negative. Otherwise, she is
clinically free. Pelvic ultrasound findings were unremarkable with the device in place. Which of
the following is the appropriate management?
Remove the device and re-insert after one month.
Combined antibiotic therapy.
Laparotomy.
Hysterosalpingogram.
Hysteroscopy.

69) A 3-year-old boy was brought to the Emergency Room with acute respiratory distress. PH was
7.26, Co2 was 55mmhg and bicarbonate was 24mmol/L. These findings indicate which of the
following?
Metabolic acidosis
Metabolic Alkalosis
Acute respiratory acidosis
Chronic respiratory acidosis
Respiratory alkalosis

70) A 26-year-old man came to the emergency department complaining of sudden onset of breath-
lessness. Examination showed that he is dyspneic at rest, with saturations of 80% on room air
and a pulse rate of 100 beats/min. His trachea is not deviated, and his blood pressure is 117/82
mm Hg. X ray revealed pneumothorax. After administering oxygen, which of the following is the
next step of management?
Intravenous furosemide
Intravenous hydrocortisone
Nebulized salbutamol
Non-invasive ventilation
Intercostal tube insertion

71) A 30-year-old man has fever (40 C°), loss of appetite and headache since 1 week. He was di-
agnosed to have typhoid fever. What are the antibiotics most commonly used in treatment of
39
typhoid? Which of the following is/are the antibiotic/s most commonly used in treatment of
Typhoid fever?
Chloramphenicol.
Ampicillin.
Fluoroquinolones.
Trimethoprim-Sulfamethoxazole.
Erythromycin.

72) A 65-year-old man presents with sudden-onset severe epigastric pain. He is a smoker and admits
to regular excessive alcohol intake. On examination, he is guarding in the epigastrium. An erect
chest X- ray shows a pneumoperitoneum. Which of the following is the most likely diagnosis?
Perforated duodenal ulcer
Perforated diverticular disease
Acute myocardial infarction
Gallbladder empyema
Ruptured aortic aneurysm

73) A 4-year-old child was brought to the office complaining of short breath and edema of both feet.
Liver span was 16 cm and hepatic tenderness was noticed. Heart sounds were weak without any
murmurs. X ray of the chest showed significant cardiomegaly. There was history of upper respi-
ratory catarrh 8 weeks ago. Which of the following is the most likely diagnosis?
Rheumatic mitral valve disease
Dilated cardiomyopathy
Fallot tetralogy
Congenital Aortic stenosis
Acute pulmonary edema

74) After 3 months of marriage, a couple (husband 38 years old and wife 31 years old), consulted
their doctor being anxious about their fertility. Their premarital assessments were unremarkable.
Which of the following is the appropriate management?
To wait until completion of one year of marriage.
Repeat pre-marital tests.
Request semen analysis of the husband.
Full clinical assessment, pelvic ultrasound and the husband’s semen analysis.
Request hysterosalpingogram.

75) A 70-year-old woman is brought to the Emergency Department with a 2-day history of rapidly in-
creasing, painless abdominal distension. The patient has a history Parkinson’s disease. No vom-
iting, no defecation for 4 days and no flatus for 2 days. On examination, the abdomen is soft and
non- tender but, grossly distended, with a tympanic percussion note. An abdominal X-ray shows
a dilated gas filled large bowel with a coffee bean appearance. Which of the following is the most
likely diagnosis?
Constipation
Rectal Cancer
Femoral hernia
40
Sigmoid Volvulus
Pseudo obstruction

76) A 36-year-old Para 5 has just delivered vaginally. After delivery of the placenta, brisk vaginal
bleeding is noticed. The uterus is soft and fails to maintain adequate contraction. Her pulse rises
to 120 beats per minute and blood pressure drops to 80/40 mmHg. Which of the following is the
FIRST step of management?
IV fluids and prepare for possible blood transfusion.
Uterotonic drugs.
Exploration of genital tract.
Bimanual uterine compression.
Hysterectomy

77) A 23-year-old man presents with prolonged nose bleeds. He has always noted easy bruising, and
ongoing bleeding after minor cuts. There is no prior history of surgery or dental procedures. His
hemoglobin is 14.5 g/dL, platelets 200,000/ml, and PT/PPT is normal. Further testing reveals
that the bleeding time is elevated; the factor VIII level is reduced, as is the ristocetin cofactor
assay. What is the most likely diagnosis?
Hemolytic uremic syndrome (HUS)
Factor 5 mutation
Antiphospholipid syndrome
Disseminated intravascular coagulation (DIC)
Von Willebrand disease (VWD)

78) A 67-year-old man complained of cough, expectoration and dyspnea of 6 years. He is a heavy
cigarette smoker. Examination revealed hyperresonance with encroachment on liver and cardiac
dullness & vesicular breath sound with prolonged expiration and expiratory rhonchi. Which of the
following is the most likely diagnosis?
COPD
Bronchial asthma
Bronchiectasis
Lung abscess
Acute bronchitis

79) A 4-year-old boy is brought to the physician with blood pressure of 135/90 mm Hg. Past medical
history is positive for three urinary tract infections. Which of the following symptoms or signs he
is likely to exhibit?
Multiple cranial nerve palsy
Headache
Hyporeflexia
Increased urine output
Right ventricular hypertrophy

80) An 8-year-old child was brought to Emergency Room by pallor. No jaundice. Complete Blood
Count showed that Hb is 7gm/dl, MCV 61, MCH 15, RDW 19, Hb electrophoresis Hb A2 3% Hb A
41
96%. Which of the following is the most likely diagnosis?
Iron deficiency anemia
B-thalassemia trait
Congenital sherocytosis
G6PD deficiency
Sickle cell anemia

81) A 39-year-old, para 3, presents with an abnormal Pap smear, on colposcopy, abnormal vascular
pattern was seen including punctuation and mosaicism. The definite final diagnosis of her condi-
tion is made by:
Histopathology
Tumor markers
Schiller iodine stain
CT scan
Surgical excision

82) Which of the following is the most common cause of primary cardiac arrest in children?
Congenital heart disease
Coronary artery disease
Head injury
Respiratory failure
Toxicologic exposure

83) A 60-year-old man complains of chest pain of 2 hours duration. He is diabetic. The patient is
sweaty, pulse is 110 b/min and Bp is 90/60. There is bilateral basal crepitation over the back of
his chest. ECG showed raised ST segment in VI till V6. Troponin is positive. Which of the following
is the most likely diagnosis?
Stable angina
Unstable angina
Acute myocardial infarction
Acute pulmonary embolism
Acute pericarditis

84) An edentulous 63-year-old woman is admitted to the ICU with inhalation injury after a house fire.
As she is being observed, she begins to have respiratory stridor. There is difficulty with bag mask
ventilation and intubation attempts. Which of the following is the most likely next step in man-
agement?
Cricothyrotomy
Corticosteroids then another trial of intubation
Trans nasal intubation
Laryngeal mask intubation
None of the above

85) A 72-year-old woman complained of 3 months of change in bowel habit including frequent di-
42
arrhoea and urgency. No blood in stool. Her weight is stable and there are no palpable masses.
Which of the following is the single most appropriate first investigation to confirm the diagnosis?
Barium enema (double contrast)
Colonoscopy
CT scan of the abdomen and pelvis
Endoanal ultrasound scan
Flexible sigmoidoscopy

86) A newborn infant 36-weeks-gestational age white male was born after 22 hrs of premature
rupture of the amniotic membranes. His weight is 2700 grams. The Apgar scores are 3 and 5. He
immediately experienced respiratory distress and cyanosis requiring ET intubation and MV with
100% O2. Vital signs are Temerature: 36.2 C, HR 195 bpm, mean BP 22 mm Hg. WBC 1500/µ L,
platelets 59,000/µL which of the following is the next most appropriate treatment for this baby?
Surfactant by aerosol
IV ampicillin and gentamicin/cefotaxime
IV steroids
IV acyclovir
HFOV

87) Which of the following signs is the most alarming for acute aortic dissection in a 51-year-old
male presenting with acute chest pain?
Hypotension:
Pallor
Pulmonary venous congestion
Unequal pulse bilaterally
Tachycardia

88) A patient sustained a laceration of the perineum during delivery, it involved the muscles of peri-
neal body but not the anal sphincter. Which of the following is the degree of this laceration?
First degree
Second degree
Third degree
Forth degree
Fifth degree

89) A 65 years-old-man presented with inability to pass urine for 8 hours. Examination revealed
tender distended suprapubic swelling, TRUS showed large symmetrical swelling of prostate with
no focal lesions and increased thickness of bladder wall. Which of the following is the most likely
cause of this condition?
Cancer prostate
BPH
Cancer urinary bladder
Urethral stricture
Diverticulum of urinary bladder
43
90) A 6-year-old boy had a sore throat and had been given antibiotics. Three weeks later, he was
brought to the office feeling feverish with nausea, vomiting and tea-coloured urine. Urine dip-
stick confirms haematuria and protein. Blood pressure is 100/60mmHg. Which of the following
is the most likely diagnosis?
Nephritic syndrome
UTI
Acute tubulointerstitial nephritis
Minimal change glomerulonephritis
Post streptococcal glomerulonephritis

91) A 33-year-old gravida 2 Para 1 (stillborn infant). She had a screening test for diabetes mellitus at
10 weeks pregnancy. One-hour post-prandial glucose test is 145 mg/dl. What should be included
in the follow-up of this patient?
No further testing and no need for treatment
Serial 3D ultrasound.
Standard glucose tolerance test.
Serial follow up with domiciliary glucose urine testing
Serial follow up with domiciliary blood sugar testing

92) Which of the following is the commonest cause of diarrhea followed by hemolytic uremic syn-
drome ( HUS)?
Shigella
Salmonella
Campylobacter
E-coli
Streptococcus

93) A 65-year-old diabetic patient is hospitalized because of acute cholecystitis. His diabetes is
controlled with metformin 850 mg twice daily. A recent hemoglobin AIC level was 6.4. Chole-
cystectomy is performed, but is complicated by postoperative pneumonia and septic shock. The
patient requires endotracheal intubation and ICU care. Blood cultures grow gram-negative rods,
and are required to maintain peripheral perfusion. Which of the following is the best method of
controlling blood sugars in this patient?
Continue metformin via nasogastric tube.
IV insulin infusion to maintain blood glucose 140 to 180 mg/dL
Sliding scale regular insulin to maintain blood glucose 80 to 120 mg/dL.
IV insulin infusion to maintain blood glucose below 100 mg/dl.
Contact endocrinology for subcutaneous insulin pump and continuous glucose monitoring

94) An eight-year-old girl develops fever 39° C and headache for two days. She receives paracetamol
suppositories. This morning the headache markedly increases in severity and she develops pro-
jectile vomiting and convulsions. On examination, conscious level is disturbed together with neck
rigidity and photophobia. Which of the following is the most likely diagnosis?
Idiopathic epilepsy
Meningitis
44
Migraine
Paracetamol toxicity
Posterior fossa tumor

95) A 30-year-old woman is admitted with a massive post-partum haemorrhage. She is shocked and
confused. Her husband informs you they are both Jehovah’s Witnesses and will not accept blood
transfusions even in life-saving circumstances. He produces a signed and witnessed document
confirming her refusal of blood products. Which of the following is the most suitable course of
action?
Follow patient’s advanced directive
Proceed under presumed consent
Written consent
Treat in patient’s best interests
Court order

96) A 23-year-old student is brought to A&E after she had a witnessed fit. History was given by a
friend as she was too drowsy. She suddenly lost consciousness after sitting on a sofa, fell to the
floor, then became rigid and started shaking all limbs vigorously. She bit her tongue and was
incontinent of urine. Which of the following is the most likely diagnosis?
Tonic-clonic seizures
Hyponatremia
Absence seizures
Hypomagnesemia
Hypocalcemia

97) A 72-year-old man ia seen in the Pre-operative assessment clinic prior to an elective thyroidecto-
my. He is currently taking Warfarin for atrial fibrillation and he asks if he needs to stop it before
his operation. Which of the following is the corrcet advise that must be delivered to him?
No need to stop
Omit morning of surgery
Stop 5 days before
Stop 10 days before
Stop 14 days before

98) A 33-year-old Para 3 comes to the physician for an annual examination. She has no complaints.
Obstetric history is significant for 3 normal vaginal deliveries with gestational diabetes during
the last 2 pregnancies. She doesn’t take any medications, Family history is significant for pater-
nal coronary artery disease. Physical examination is unremarkable. Which one of the following
investigations should this patient perform?
Chest X-ray every 3 years.
Coronary angiography every 3 years.
Fasting glucose testing every 6 months.
Mammography every 3 years.
Pelvic ultrasound every 3 years.

45
99) A man has the risks and potential benefits of his cholecystectomy explained to him. Which ethi-
cal principle is best described here?
Gillick competence
Assent
Consent
Autonomy
Capacity

100) A 20-year-old man with asthma has been coughing for the past couple of days. He has devel-
oped shortness of breath in the past few hours and is finding it difficult to talk. His regular in-
halers have not helped him. His heart rate is 112 bpm and respiratory rate 30 breaths/min. The
severity of his asthma is which of the following?
Acute severe
Brittle
Life-threatening
Moderate exacerbation
Near fatal

6 June 2023 real exam

1) You see a 72-year-old man in your pre-operative assessment clinic prior to an elective open in-
guinal hernia repair. He is currently taking clopidogrel following a coronary stent placement 18
months ago, and he asks you if he needs to stop it before his operation, which of the following is
your advice?
No need to stop
Omit morning of surgery
Stop 3 days before
Stop 7 days before
Stop 14 days before

2) Which of the following statements regarding clostridial wound infections and their management
is NOT true?
Clostridia are anaerobic, terminal, spore bearing, Gram-positive bacteria.
Thin, brown and sweet-smelling exudate is seen in gas gangrene.
Necrotic and foreign material in wounds increase risk.
The spores are widely spread in soil and manure.
The signs and symptoms are due to the endotoxins.

3) A 56-year-old man is reviewed in the Cardiology outpatient clinic following a myocardial infarc-
tion one year previously. During his admission he was found to be hypertensive and diabetic. He
complains that he has put on 5kg in weight in the past 6 months. Which of his medications may
be contributing to his weight gain?
Gliclazide
Losartan
Clopidogrel
46
metformin
Simvastatin

4) A 36-year-old female presents to the ED with a chief complaint of finger pain and swelling. She
had previously been treated for an acute paronychia a few months ago, but states it “won’t go
away.” Examination reveals a dystrophic nail, loss of the cuticle, as well as an area of tender, ery-
thematous swelling along the proximal nail fold, without abscess formation. This chronic parony-
chia is best treated with which of the following?:
Topical corticosteroids
Oral antibiotics
Oral antifungals
Warm water soaks
Oral antivirals

5) A 45-year-old female presents with depressed mental status, hypothermia, hypotension, and
bradycardia. She has not been taking her thyroid replacement therapy. Which of the following
findings is most likely to be present?
Hyperglycemia
Hypernatremia
Hypercapnia
Hyper-reflexia
Polycythemia

6) A female 30 years old had an IUD and presented to the hospital with severe lower abdominal pain.
ultrasound reveals fluid in Douglas pouch and IUD in place which of the following is the Next step
in management?
Bed rest
Antibiotics
Analgesics
Remove IUD and give antibiotics
assurance

7) A 7-year-old boy presents with fever 39.2 and right sided abdominal pain. His mother gives histo-
ry of poor appetite and cough for 2 days. On examination HR: 120/minute, RR: 45/minute, breath
sounds are diminished with bronchial breathing over right lower lung zone and abdomen is dif-
fusely tense. which of the following is is the initial diagnostic test?
Abdominal CT
Abdominal ultrasound
Chest Xray
Uver function tests
examination under anesthesia

8) A 52-year-old diabetic man presented by a lump in the lower abdomen and scrotum, marked
abdominal distension and he did not pass stool for 2 days & vomited 3 times in the way to hos-
pital. On examination temperature was 38.2, there was generalized abdominal tenderness & the
47
swelling was tender and irreducible. X ray shows multiple air-fluid levels. Random blood glucose
was 385 mg/dl. Which of the following is the next step in the management ?
Correction of hyperglycemia followed by surgery in the next day.
Urgent surgical interference.
Preparation for elective surgery
Admission NPO,NGT,observation for intestinal obstruction.
Manual reduction and discharge.

9) A 2-hour-old full-term neonate is noted by the nurses to have episodes of cyanosis. When they try
to feed him, his oxygen level drops to 60%. But when he is stimulated and cries, his oxygen levels
increase above 90%. Which of the following should be done to diagnose this case?
Bronchoscopic evaluation of palate and larynx.
Echocardiogram.
Hemoglobin electrophoresis.
Passage of catheter into nose.
ultrasound

10) A 28 years pregnant woman at 22 weeks gestation was discovered to have asymptomatic bacter-
uria. Which of the following is the suitable action to be given?
Expectant management
Induction of labor.
Antibiotics
Diuretics.
Intravenous hydration

11) A 16-year-old primigravida a term, not in labour, presented to the ER with a sudden onset of
continuous lower abdominal pain with no other complaints. Examination revealed rapid pulse,
no fetal heart tones, low BP, and a tender hard uterus. which of the following is the Most likely
diagnosis
Abruptio placentae
placenta previa
Amniotic fluid embolism
Supine hypotensive Syndrome
normal labour

12) A32 year old infertile woman presented with long standing history of irregular cycles, obesity
and hirsutism and acne, which of the following is the first Laboratory investigation for this pa-
tient.
Testosteron Serum and DHEAS
TSH and Prolactin
17 hydroxy progesteron
All of the four options
pelvic and abdominal ultrasound

48
13) Sign’ that refers to a spontaneous cessation of inspiration on palpation of the hypochondrium
just below the right costal margin. What is the eponymous sign or law being described?
Courvoisier’s
Rovsing’s
Murphy’s
Grey Turner’s
Battle’s

14) A female patient, 31 years old, complained of and increased abdominal girth, after delivery.
Abdominal examination revealed ascites by shifting dullness. Laboratory investigations reported
marked proteinuria and hypoalbuminemia. Abdominal ultrasonography revealed fatty liver and
ascites. Which of the following is the most likely cause of ascites?
Nephrosis
Poncreatic ascites
Liver cirrhosis ond oscites
Abdominal tuberculosis
tubercolosis

15) A 14-month-old infant presents with a 1-day-history of low grade fever. On the next day a vesic-
ular rash develops over the trunk and abdomen. The rash is itchy. Which of the following is the
most common complication for that disease?
Encephalitis
Neuritis
Skin infection
Subacute sclerosis panencephalitis
Rheumatic fever

16) which of the following is a character of angioedema?


usually causes pleural effusion
May cause suffocation
Treated by topical steroids
treat with antibiotics
just reassure.

17) which of the following is The ideal infusion fluid for correction of hypokalaemic alkalosis due to
pyloric obstruction is:
Normal saline
Potassium chloride in 59% glucose
Ringer’s solution
Ammonium chloride
Glucose-saline

18) 65-year-old woman is noted to have Suspected Uterine Fibroid on Physical Examination. Over
the Course of 1 year, she is noted to have enlargement of Uterus approximately from 12 weeks to
49
Size 20 weeks. Which of the following is the best management?
Continued careful examination
Monitoring with ultrasound examination
Exploratory laparotomy with hysterectomy
Progestin therapy
nothing apart from assurance

19) A 26-year-old man has had a 6-month history of non-bloody diarrhea, malaise, recurrent ab-
dominal cramps, and temperature to 38.50C. At this time, he is afebrile. Examination reveals oral
ulcers and a palpable, ill-defined mass in the right lower quadrant of the abdomen. Palpation
causes local tenderness without guarding. Which of the following is the most likely diagnosis?
Appendicular mass
Chronic appendicitis
Crohn’s disease
Intestinal lymphoma
Ulcerative colitis

20) A 4-year-old girl presents with coryzal symptoms, cough, fever & abdominal pain. There is no
dysuria, & urine dipstick is unremarkable. On examination, the pain is in the left upper quadrant
but is not severe. Which of the following is the most likely diagnosis?
Urinary tract infection (UTI)
Appendicitis
Pyelonephritis
Left basal chest infection
Mesenteric adenitis

21) 18-year-old female at 7 weeks’ gestation confirmed by reliable LMP, complains of 2-day history
of Vaginal Spotting and lower abdominal pain Physical examination reveals 4-week Sized uterus.
On U/S No intrauterine gestational sac in noted. which of the following is the next step in man-
agement?
Observation
Do B-hcG level
Give her Kapron to stop bleeding
Ask her to do U/s after 2 days
Dilation and evacuation

22) A male patient 45 years old, known to be IBD patient maintained on azathioprine referred to
clinic due to abnormal lab results as follow: high ALP, GGT, conjugated bilirubin, Transaminases
(AST/ALT) are within normal limits, high Cholesterol and high IgM. Ultrasound revealed Irregu-
lar diameter of the bile duct with diffuse thickening of the wall of the common hepatic and bile
ducts. Which of the following is most likely diagnosis?
Autoimmune hepatitis
Cancer head of pancreas
Primary sclerosing cholangitis
Primary biliary cholangitis
50
acute hepatitis

23) A 30-years old female presents with right hypochondrial pain for 2 days associated with fever.
The pain started mild, now severe radiated to the back and right shoulder, exacerbated by fat-
ty food and relieved by analgesics. On examination: Temp. 38.6°C, no jaundice and abdominal
palpation relieved tenderness & guarding in right hypochondrium, which of the following is the
most likely diagnosis?
Chronic hepatitis
Acute pancreatitis
Acute cholecystitis
Acute appendicitis
chronic colitis

24) A 60-year-old woman has had a 6 month history of bloody diarrhea. She presents to the ER with
severe abdominal pain and constipation. On examination, her abdomen is distended and tender
on the left side. Rectal examination reveals small amount of faeces. Blood tests shows Hb 11.0
g/dl TLC 15.6 x 109/1. platelets 450 x 109/1. Sodium is 139 mmol/l (N: 135-145), potassium
is 4.6 mmol/l (N: 3.5-5), urea 25 mg/dl, creatinine 1.3 mg/dl and C-reactive protein (CRP) is 75
mg/l. What is the next best of further investigation?
Colonoscopy
CT of the abdomen
Magnetic resonance imaging (MRI) of abdomen
Sigmoidoscopy
X-ray of abdomen

25) A newborn noticed to have chocking attacks immediately in starting bottle feeding associated
with cyanosis. On examination his heart was normal, but his lungs show bilateral crepitations.
After two days he was worse and hospitalized. CXR revealed aspiration pneumonia, and he was
put in the ventilator. which of the following is The nest step to reach a diagnosis?
Arterial blood gases.
Barium meal.
Echocardiography.
Radio-opaque nasogastric tube
examination under anesthesia

26) A40 year G5P5 woman with history of enlarged uterus Complains of menorrhagia and anemia.
Examination reveals irregular midline mass approximately 4 weeks’ size. The mass is diagnosed
by imaging as intramural myoma. Which of the following is the first line of Management?
Tranexamic acid and iron
Medroxy progesterone, tranexamic acid, and iron
Oral contraceptive pills and iron
Methotrexate and iron
hysterectomy

27) A 64-year-old man presents with a 2-day history of abdominal pain which he describes as con-
stant, dull and around his umbilicus and occasionally migrating to his groin. He has a body mass
51
index (BMI) of 27 and a past medical history of poorly controlled hypertension. Abdominal exam-
ination reveals a pulsatile and expansile mass just below the umbilicus. What is the most appro-
priate screening investigation?
Abdominal ultrasound
Abdominal x-ray
Computed tomography (CT) scan of the abdomen.
Abdominal magnetic resonance imaging (MRI) scan
Angiography

28) which of the following is expected to be found in the assessment of a child with CP?
Regression in previously acquired milestones
Seizures
A macular cherry red spot on fundus examination
Persistent metabolic acidosis
Lens dislocation

29) A 5-year-old girl is brought in to see you by her mother because the girl has developed complete
breasts and a new pubic hair, Evaluation demonstrates a pubertal response to GnRH Simulation
test and a prominent increase in LH pulses during sleep. These findings are characteristic of pa-
tients with which of the following ?
premature thelarche
latrogenic Sexual precocity
Central precocious puberty
Granulosa cell tomours
normal menarch

30) A 45-year-old female complains of lower abdominal pain and vaginal discharge. On examination,
there is cervicitis along with a mucopurulent cervical discharge. The Gram smear of the discharge
shows presence of abundant pus cells, but no bacteria. Which of the following is the best ap-
proach to isolate the possible causative agent?
Culture on chocolate agar supplemented with hemin
Culture on vero cell lines
Culture on McCoy cells
Culture on a bilayer human blood agar
None of the above

31) A 25-year-old P0+2 aborted 5 months back at 12 weeks of a gestation. She has had a surgical
evacuation. She has not got her periods yet. Urine pregnancy test is negative. Estrogen proges-
terone withdrawal test is negative. What is the most likely diagnosis?
Pituitary failure
Ovarian failure
Anovulation
Asherman syndrome
imperforate hymen
52
32) A 34-year-old man is extracted from an automobile after a motor vehicle collision. The patient
has an obvious deformity of his right thigh consistent with a femur fracture. Upon closer exam-
ination of the right thigh, there is bone visible through an open wound. Which of the following is
the appropriate management of his open femur fracture?
early irrigation and debridement, IV antibiotics, and cast or splint placement
early irrigation and debridement, IV antibiotics, and internal or external fixation
early irrigation and debridement, IV antibiotics, compartment decompression, and internal or
external fixation
intravenous (IV) antibiotics and cast or splint placement
IV antibiotics and internal or external fixation

33) A 31-year-old man is brought to the emergency room following a motor car accident in which his
chest struck the steering wheel. Examination reveals stable vital signs, but the patient exhibits
multiple palpable rib fractures and paradoxical movement of the right side of the chest. Chest
X-ray shows no evidence of pneumo-or hemothorax but a large pulmonary contusion is develop-
ing. What is the proper management for this case?
immediate operative stabilization.
none of the all
stabilization of the chest wall with sandbags.
stabilization with towel clips.
tracheostomy, mechanical ventilation, and positive end-expiratory pressure.

34) A 6-year-old boy is brought to the emergency room with a 3-hour history of fever 39.5°C and
sore throat. The child appears anxious and toxic. He has mild inspiratory stridor and he is drool-
ing. He is sitting on the examination table leaning forward with his neck extended. Which of the
following is the most appropriate immediate management of the patient?
Admit the child and place him in a mist tent.
Examine the throat and obtain a culture.
Obtain an arterial blood gas and start an IV line.
Prepare to establish an airway.
non of the all answers

35) 1 year. An infant can wave bye-bye, points to body parts and feed with spoon. He walks unsup-
ported, but still unable to give full name, age and sex. Which of the following is the most likely
age of the infant?
1 year.
2 years.
3 years.
4 years.
5 years

36) Abdominal ultrasonography of 23 years old, male patient with abdominal enlargement revealed
moderate ascites with adhesions. Serum-ascites albumin gradient (SAAG) was <1.1 g/dL. Which
one of the following is the best diagnostic tool?
Laparoscopy

53
Abdominal computed tomography
Gastroscopy
Ascitic fluid cytology
exploration

37) A 70-year-old man had a right hemicolectomy for cecal carcinoma 4 days ago. He is now having
abdominal distention and recurrent vomiting. He has not opened his bowels since surgery. There
are no bowel sounds. White blood cells is 9000/dl, Temperature is 37.5°C. What is the single
most appropriate next management?
antibiotic therapy IV
glycerin suppository
laparotomy
nasogastric tube suction
TPN

38) A 5-year-old child has a 6-month history of increasing difficulty in walking associated with
urinary incontinence. Physical examination shows an alert child with increased deep tendon
reflexes, clonus and bilateral Babinski reflexes in the lower extremities with grade 3/5 weakness
symmetrically in all extremities. Which of the following is the most likely diagnosis?
Guillain-Barré syndrome.
Medulloblastoma.
Myasthenia gravis.
Spinal cord tumor
myalagia

39) A 5-year-old boy developed acute itchy skin rash characterized by wheals covering a large area of
his body. His temperature was 37.2°C, what is the most probable cause of his rash?
Measles
Hay fever
Psoriasis
Urticaria
Familial Mediterranean fever (FMF)

40) This is a 7-month old boy who is not growing well. His birth weight was 3.5 kg and his current
weight is 4.5 kg (less than the 5th percentile). Mother states that he drinks 60 ml of human-
ized of infant formula every 4 hours, she also feeds him a small amount of rice cereal he was 5
months old. Which sign of the following you are expecting to find in examination?
Ascites.
Hepatomegaly.
Loss of fat from the abdomen.
Microcephaly
non of the all answers

41) which of the following is not a feature of Genetic short stature ?


54
Short parents
Short as an adult
Advanced bone age
Being a normal variant
Proportionate

42) A 45-year-old woman presents to the emergency room with abdominal pain and vomiting for 2
days. She is passing flatus but has not passed stools since yesterday. On examination the abdo-
men is tender with marked distension. There is a tender swelling above the umbilicus which is
crescentic in shape. The swelling is irreducible, dark in colour and does not give an impulse on
coughing. The pulse rate is 108 bpm, blood pressure 90/50 and temperature is 38.0°C. What is
the immediate management?
Early fluid resuscitation
Preoperative detailed investigation to detect the cause
Proper examination of content
Resection of devitalized partes
Surgical interference

43) An 8-year-old girl presents with weight loss, diarrhea & abdominal pain. On examination, you
note the presence of aphthous ulcers & generalized abdominal tenderness. She is off her food
but drinking normally, & her energy levels are low. What is the most likely diagnosis?
Anorexia nervosa
Crohn’s disease
Diabetic Ketoacidosis (DKA)
Recurrent apthous stomatitis
Ulcerative colitis (UC)

44) A female patient, aged 25 years old, complained of weight loss and progressive increase of ab-
dominal girth for two months. Abdominal ultrasonography revealed moderate ascites with adhe-
sions. Which of the following is the best initial diagnostic tool?
Complete blood count
Laparoscopy
Abdominal radiography
Ascitic fluid cytology
karyotyping

45) A 7-year-old girl presents with a tender and swollen right knee joint, as well as fever. Recently,
she developed arthritis of the left wrist. Which of the following modified Jones criteria this pa-
tient has?
1 major & 1 minor.
2 major.
2 major & 1 minor.
2 minor.
all of the four options
55
46) which of the following organisms can cause Rheumatic fever?
Streptococcus
RSV
Pneumococcal
Meningococcal
H influenza

47) A 45-year-old female presents with depressed mental status, hypothermia, hypotension, and
bradycardia. She has not been taking her thyroid replacement therapy. Which of the following
findings is most likely to be present?
Hyperglycemia
Hypematremia
Hypercapnia
Hyper-reflexia
Polycythemia

48) In A case of acute cholecystitis; which of the following is specific criterion for diagnosis?
Liver function tests are minimally affected
Abdominal US a thick-walled GB with peri-cholecystic fluid
Plain AXR show left radiopaque shadow
CBC showed increased TLC
increased ketone bodies in urine

49) A 64-year-old man suffers an anterior myocardial infarction. A few hours later, his pulse rate is
noted to be 46/minute and his blood pressure 94/59 mm Hg. He is short of breath and has slight
central chest pain. The monitor showed sinus bradycardia. What would be your choice of man-
agement?
Insertion of temporary pacing wire
Intravenous isoprenaline
Intravenous atropine
Intravenous adrenaline
Oral salbutamol

50) A 14-year-old boy was referred because of his tall stature. His height was more than the 95th
percentile. He had bilateral small testes, no public or axillary. Which of the following is The cause
of such a condition ?.
Delayed puberty
Down syndrome
Klinefelter syndrome
Marfan syndrome
non of the all answers

51) An edentulous 63-year-old woman is admitted to the ICU with inhalation injury after a house fire.
As she is being observed she begins to have respiratory stridor. There is difficulty with bag mask
56
ventilation and Intubation attempts, which of the following is the next step in management ?
Cricothyrotomy
Corticosteroids then another trial of intubation
Trans nasal intubation
Laryngeal mask intubation
None of the above

52) During the evaluation of a 30-year-old female complaining of infertility, a hystero salpingogram
showed evidence of Asherman syndrome. which one of the following symptoms would expect
this patient to have?
Oligomenorrhea
Menorrhagia
Metrorhagia
Hypomenorrhea
Dysmenorrhea

53) If you found a child in coma with pin pointed pupil what will you suspect?
Barbiturates intake
Hypothermia
Third n lesion
Severe hypoxia
Seizures

54) A 26-year-old patient came to the outpatient with a 6-week history of fever, bloody diarrhea and
weight loss. On examination, he is clinically anemic and has aphthous ulcer of the mouth and
mild tenderness of the abdomen. Sigmoidoscopy shows transmural inflammation and non-ca-
seating granulomas of the colon and rectum. If the patient also complains of that lesion on his
shins: What is the diagnosis of this skin lesion?
Erythema nodosum
Pyoderma gangerosum
Dermatitis hepetitformis
Eczyma
Atopic dermatitis

55) you are called to review a 44-year-old woman who is admitted with known gallstones. She is
normally on omeprazole for gastritis. The patient complains that since admission she has started
to feel itchy and that her pain becomes worse after some of her daily medications. The patient is
not jaundice and latest bloods are all normal. The doctor reviews the patient’s drug chart. Which
of the following medications could be causing her symptoms?
Buscopan
Ibuprofen
Morphine
Omeprazole
Paracetamol
57
56) which of the following coincides with WHO definition of anemia with pregnancy?
Hemoglobin level below 11 g/dL
MCV value below 80 fl
Hematocrit value less than 25%
MCHC below 33%
Reticulocytic count less than 2%

57) A 24-year-old man is referred to the neurology clinic with an 8-month history of headaches. He
complains of feeling low for the past few months as well. He describes a generalized head pain,
feeling like a tight band around the head and it feels like a clamp squeezing the head. He denies
any other ocular or nasal symptoms and you find examination to be normal. What is the most
likely diagnosis?
Chronic sinusitis
Migraine
Tension-type headache
Trigeminal neuralgia
Cluster headache

58) 18 year old female at 7 weeks gestation by LMP complains of 2 day history of Vaginal spotting
And lower abdominal pain Physical examination reveals 4 week Sized uterus. On U/S No intra-
uterine gestational sac in noted What is the next step management?
Observation
Do B-hcG level
Give her Kapron to stop bleeding
Ask her to do U/s after 2 days
Give methotraxate

59) A 55-year-old man presents with locking and clicking of his knee for several days. He twisted it
several weeks ago and did not seek medical care at the time. Physical examination demonstrates
no knee instability or patellar tenderness. His knee clicks with flexion and occasionally gets
locked just before full extension. Which of the following structures is most likely injured?
Anterior cruciate ligament
Posterior cruciate ligament
Medial meniscus
Patella tendon
Medial collateral ligament

60) 22 years old patient married since 3 years comes to outpatient clinic with a history of inability to
conceive. her menstrual history is unremarkable in terms of regularity. Since 2 years she started
to complain of severe central lower abdominal pain that occurs 2 days before, continue during
the menstrual period and 2 days after its end. Pelvic U/S. is painful and shows empty cavity,
Right ovary shows unilocular-complex cyst, which of the following is the Most Probable diagno-
sis?
Ovarian Cancer
Functional Ovarian cyst
58
Dermoid cyst
Endometriosis
ectopic pregnancy

61) An injury to a major artery in an extremity is suspected in male patient 40-year-old after motor
car accident. The surgeon decided to explore the femoral and popliteal vessel in spite of presence
of distal pulses to the injury: What is the rationale behind the surgeon’s decision?
hematoma compressing proximally on the main artery.
significant injury to adjacent long bones.
significant injury to adjacent motor nerve trunks.
subsequent development of a compartment syndrome and the need for fasciotomy.
the presence of palpable distal pulses does not reliably exclude significant arterial injury.

62) A 35year-old woman with three children and using intrauterine device for contraception. She
presents with irregular menstrual cycles at intervals anywhere from 33 to 90 days. Menstrual
flow is usually heavy, and she has noted increasing hirsutism. The medication MOST SUITABLE
for her is:
Clomiphene citrate (Clomid)
Combined oral contraceptives
Antiprostaglandin
progesterones
HCG

63) A 72-year-old woman presents to the ED with acute left lower quadrant abdominal pain. Her past
medical history is significant only for mild hypertension. On exam, she is febrile to 38.2 °C with
focal rebound tenderness in her left lower quadrant. She has a WBC of 12 000 /mm3. Abdominal
CT imaging reveals a thickened sigmoid with a 3.0 cm x 3.0 cm peri-sigmoid colon abscess with
a very small pocket of extraluminal air in the pelvis. which of the following is The next step in
management ?
Exploration, sigmoidectomy with descending end colostomy (Hartmann’s procedure)
Exploration with sigmoidectomy, primary colorectal anastomosis
Exploration with sigmoidectomy, primary colorectal anastomosis, and loop ileostomy
Resuscitation, broad spectrum IV antibiotics and percutaneous drainage of the abscess
Resuscitation, broad spectrum IV antibiotics, colonoscopy for likely colon carcinoma on this ad-
mission

64) A 68-year-old woman complains of sudden onset of severe pain in her right thigh, calf and leg
of 6 hours duration. She has lost sensations from the mid-thigh distally, cannot move her foot
or knee, her skin feels very cold and pale. There are no pulses felt over the femoral and popliteal
arteries. She gives a history of a myocardial infarction 6 weeks ago. What is the appropriate diag-
nosis?
abdominal compartment syndrome (ACS).
acute femoro-popliteal embolus
diabetic gangrene
dissecting aortic aneurysm
59
popliteal artery aneurysm

65) What is the best management of a newly agitated patient in the ward?
Give him midazolam
Give him carbamazepine
Give him ketamine
Give him lamotrigine
Give him haloperidol

66) A 63-year-old woman is admitted to the hospital with severe abdominal pain of 3-hour duration.
Abdominal examination reveals board-like rigidity, guarding, and rebound tenderness. Her blood
pressure is 90/50 mm Hg, pulse 110 bpm (beats per minute), and respiratory rate is 30 breaths
per minute. After a thorough history and physical, and initiation of fluid resuscitation, which of
the following is the diagnostic study that should be performed?
Supine abdominal x-rays
Upright chest x-ray
Gastrograffin swallow
Computerized axial tomography (CAT) scan of the abdomen
Abdominal sonogram

67) A 26-year-old G1 PO has a positive glucose challenge test at 28 weeks gestational age (GA).
she is obese but without previous health problems. Follow-up testing confirms the diagnosis of
gestational diabetes mellitus (GDM). The patient asks what this diagnosis will mean for her baby.
Which of the following can be a significant consequence of poorly controlled GDM to the fetus?
Hyperglycemia
Macrosomia
Cataracts
Cerebral edema
Cephalohematoma

68) A 25-year-old woman was previously admitted to the orthopaedic ward with an ankle fracture
which was treated with a plaster cast. When the cast is removed, the patient is unable to dorsi-
flex her foot. All other leg movements are maintained. Which nerve has most likely been affect-
ed?
Common peroneal nerve
Obturator nerve
Sciatic nerve
Sural nerve
Tibial nerve

69) A 29-year-old female is noted to have an elevated bilirubin during a viral illness. Gilbert’s syn-
drome is suspected. Which of the following is elevated in Gilbert’s syndrome?
Conjugated bilirubin
Unconjugated bilirubin
GGT
60
Alkaline phosphatase
Reticulocytes

70) A 42-year-old obese mother of 5 children comes with severe upper abdominal pain mainly in the
right side and referring to the right shoulder with a temperature 37.8°C What is the most proba-
ble diagnosis?
biliary colic
cholecystitis
ectopic pregnancy
hepatitis
ovarian torsion

71) An 18-year-old G2,p 1+0, presents for her first OB visit at 10 weeks. she reports that the first
day of her menstrual period was May 7. her menstrual history was unremarkable. Which of the
following is the patient’s estimated date of delivery?
February 10 next year
February 14 next year
December to same your
December 14 Some year
February 7 next year

72) A 40 years old, male patient presented by progressive abdominal distension and bilateral lower
limb swelling for one month, following an episode of hematemesis and melena. Examination re-
vealed jaundice, palmar erythema, bilateral lower limb edema, ascites by transmitted thrill, and
dilated abdominal veins. Which of the following is the most likely diagnosis?
Abdominal tuberculosis
Pancreatic ascites
Liver cirrhosis
Malignant ascites
cancer ovaries

73) A 5-year child is admitted to the ward for investigation of glomerulonephritis, as he has peripher-
al oedema and blood in his urine, he had been treated with antibiotics for tonsilites three weeks
ago. He is otherwise well. With no rashes, joint pain or abdominal sympotoms. what is the most
likely cause of his glomerulonephtotis? select one answer only
Post-streptococcal glomerulonephritis (PSGN)
Berger’s disease (IgA)
Haemoltic uraemic syndrom (HUS)
purpura (HUP)
Henoch-Schohelein
Goodpasture syndrom

74) which of the following is The “time out” in surgery/


includes shaving of the surgical area the evening before the operation.

61
it is done inside the operating room for ensuring patient identity, type and side of the operation.
it is the recording of the time from the start of the operation till its end.
it is the time where the surgical site marking is done.
the purpose is to check for any abnormality in the patient’s preoperative investigations.

75) A 35-year-old male patient undergoes an elective thyroid surgery. The patient is not diabetic nor
suffering from any systemic illness. How can you describe his wound?
class I, Clean
class II, Clean/contaminated
class III, Contaminated
class IV, Dirty
wound to heal by secondary intention

76) Follicle-stimulating hormone is elaborated by which of the followings?


Chromophobe cells of the adenohypophysis
Basophilic cells of the adenohypophysis
Acidophilic cells of the adenohypophysis
Folliculargranulosa
Follicular cumulus

77) which of the following is NOT a mechanism of action of ursodeoxycholic acid (The active ingredi-
ent is a hydrophilic bile acid)?
Slows progression of the disease and improves clinical symptoms
Hepato protective; antiapoptotic
Suppresses immune reactions that promote disease progression
inhibits synthesis of cholesterol
treat anemiare

78) Which of the following symptoms suggest a foreign body in the nasal passage?
Severe pain and high fever
Clubbing
Mucopurulent malodorous discharge
Crepitations
Wheezes

79) A 30-year-old executive learns that he has a duodenal ulcer. His gastroenterologist prescribes
and outlines medical therapy. The patient worries that if medical therapy fails he may need sur-
gery. Which of the following is the best indication for elective surgical therapy for duodenal ulcer
disease?
An episode of melena
Repeated episodes of pain
Pyloric outlet obstruction due to scar formation from an ulcer
Frequent recurrences of ulcer disease
62
Referral of pain to the back suggestive of pancreatic penetration

80) A 40-year-old man is being investigated for an enlarging painless lump in the right testicle. Blood
results show a significantly raised alpha fetoprotein and a normal beta-hCG. Which of the follow-
ing testicular tumours is most likely to be the cause of his symptoms?
Choriocarcinoma
Leydig cell tumour
Seminoma
Testicular lymphoma
Yolk sac carcinoma

81) A25 year old patient presents for contraceptives. During counseling and assessment, which of
the following is a contraindication to IUD insertion?
history of recent Pelvic inflammatory disease
Previous pregnancy with IUD
history of vulvovaginitis
Chorio- amnionitis in Previous pregnancy
history of preterm labour

82) A 34-year-old woman is brought in following a road traffic collision, she is suspected to have
intra- abdominal injuries and there is bruising over his trunk. On arrival, he has a pulse rate of
138/min and his blood pressure is 80/50 mmHg. Which of the following fluids would raise the
patient’s blood pressure rapidly?
5% dextrose
Fresh frozen plasma
Packed red cells
Mannitol
Normal saline

83) According to Glasgow coma scale for a child 2 years old, what score will you give for a child with
eye opening to sound?
3
5
7
9
10

84) A 45-year-old woman with Crohn’s disease and a small intestinal fistula develops tetany during
the second week of parenteral nutrition. The laboratory findings include normal Na, K, CL, HCO3
and, Ca, Mg: 1.2 mg/dL (1.8-2.6 mg/dL), PO4: 2.7 mg/L (2.7-4.5 mg/dl, serum Albumin 2.4 g/
dL (3.5-5.0 g/dL), arterial blood gas sample reveals a pH of 7.42 (7.35-7.45), PCO2 of 38mmHg
(35.0-50.0 mmHg), and PO2 of 84 mmHg (85-100 mmHg). What is the cause of the patient’s
tetany?
essential fatty acids deficiency.
focal seizure
63
hyper-ventilation
hypocalcemia.
hypomagnesemia.

85) A fully heparinized patient develops a condition requiring emergency surgery. After stopping the
heparin. What else should be done to prepare the patient?
immediate administration of protamine 5 mg for every 100 units of heparin most recently admin-
istered.
immediate fresh frozen plasma.
nothing, if the surgery can be delayed for 2-3 hours.
transfusion of 10 units of platelets.
vit. K intramuscular injection

86) which of the following is the Commonest acyanotic heart disease?


ASD
VSD
PDA
AS
MS

87) A 75-year-old male patient is brought to emergency room with sudden inability to pass urine. The
patient gave history of frequency & micturition over the last few years which makes him wake up
2-3 times/night. Abdominal examination showed supra-pubic swelling. What is the diagnosis of
this condition?
benign prostatic hyperplasia
cancer prostate
pyelonephritis
traumatic urethral tear
urethral stricture

88) A 55-year-old female patient underwent pelvic surgery. On the 10th day postoperative, she de-
veloped severe shortness of breath with chest tightness. Examination revealed pulse 110 bpmin.,
BP 80/50 mmHg with central cyanosis. Chest examination were unremarkable. Which of the
following is the most suitable treatment to start?
Antithrombotic Enzymes
Clopidogrel
IV fluids
Aspirin
Warfarin

89) Which of the following is NOT A manifestation of pancreatic insufficiency?


Hypertriglyceridemia
Steatorrhea
Hyperglycemia
64
Normal blood glucose level
weight loss and diarrhea

90) Neonatal mortality remains responsible for the majority of under 5 child mortalities all over the
world. What is the main cause of neonatal mortality globally?
asphyxia
hyperbilirubinemia
post maturity
prematurity
sepsis

91) A 45-year-old man presents with fatigue, arthralgia, impotence and increased skin pigmentation.
O/E he has enlarged firm liver. Blood test reveal deranged liver function tests and impaired glu-
cose tolerance. What is the most appropriate initial investigation strategy?
Abdominal ultrasonography
CT scan
Uver biopsy
Serum iron + total iron binding capacity
Transferrin saturation + ferritin

92) A 70-year-old hypertensive man, heavy smoker with a 3-month history of backache presents
with acute excruciating epigastric pain radiating to the back. O/E: blood pressure is 80/50mmHg,
pulse is 130bpm (right femoral pulse is absent, while left femoral pulse is weak), marked abdom-
inal tenderness and rigidity. What is the cause of his abdominal pain?
Acute appendicitis
Acute pancreatitis
Ascending cholangitis
Acute MI
Ruptured abdominal aortic aneurysm

93) Which of the following IS NOT a predisposing factor for premature labor?
Maternal age older than 35 years
Smoking more than 10 cigarettes per day
Exposure to diethylstilbestral (DES) in utero with a documented uterine structural abnormality
More than four previous deliveries
Twin gestation

94) A 30-year-old woman presents with colicky lower abdominal pain, vomiting, distention, fever,
leukocytosis and an abdominal ultrasound that showed pelvic collection. What is likely the diag-
nosis?
amoebic colitis,
complicated Meckel’s diverticulum
perforated appendicitis.
salpingitis.
65
ulcerative colitis

95) A 24-years-old female hairdresser presented with a 5-day history of jaundice, right upper quad-
rant discomfort, and general malaise. Her stool was of normal colour and not pale, nor did she
have dark urine. She had no past medical, significant drug or family history. On examination she
was jaundiced, without lymphadenopathy, peripheral oedema, or finger clubbing. There were no
signs of chronic liver disease or portal hypertension. She had a liver edge, palpable 3 cm below
the costal margin, which was firm, smooth and non-tender. Bilirubin 56µmol/L: (mainly direct),
ALT 2858 IU/LT, ALP 626 IU/L, albumin 40g/L Ultrasound scan of the abdomen: The liver was ho-
mogeneous and mildly enlarged, no gallstones, but there was slight thickening of the gallbladder
wall. The pancreas, bile duct, kidneys, appeared normal. which of the following is the most likely
diagnosis?
Autoimmune hepatitis
Primary biliary cirrhosis
Primary sclerosing cholangitis
Acute cholecystitis
primary bilharziasis

96) A 9 months child is brought to the emergency room with an irreducible firm swelling which
descended into the left groin when the child has been crying. On examination, both testicles are
palpable in the scrotum. What is the appropriate management strategy?
elective herniotomy
elective hemiotomy + orchidopexy
emergency herniotomy
emergency herniotomy + orchidopexy
reassurance

97) In a patient with peripheral arterial disease, which one of the following features suggests the
worst prognosis?
Claudication distance of 50-100 m
Night pain
Diminished pedal pulses
Ankle brachial pressure index (ABPI) ratio of 0.8
Bruit over femoral artery

98) A female patient 59-year-old presented with attacks of hematemesis, on examination pulse rate
was 118 bpm., blood pressure 90/55 mmHg, nasogastric tube was inserted and revealed active
bleeding then wash is started. What is the best option for this patient’s management after initial
resuscitation?
admission and preparation for elective surgery
medical home treatment
ultrasound to detect state of the liver cirrhosis
urgent refer for surgery
urgent endoscopic work up

99) A 36-year-old female presents to the ED with a chief complaint of finger pain and swelling. She
66
had previously been treated for an acute paronychia a few months ago, but states it “won’t go
away.” Examination reveals a dystrophic nail, loss of the cuticle, as well as an area of tender, ery-
thematous swelling along the proximal nail fold, without abscess formation. This chronic parony-
chia is best treated with which of the following?:
Topical corticosteroids
Oral antibiotics
Oral antifungals
Warm water soaks
Oral antivirals

100) A 60-year-old female with a history of diabetes and hypertension presents with new-onset ver-
tical diplopia. His examination reveals normal pupils, mild right sided ptosis, and slight inferior
deviation of his right eye at rest. At times the right eye appears to “get stuck” after being abduct-
ed in downward gaze. Which of the following is most likely affected?
Cranial nerve ii
Cranial nerve iii
Cranial nerve IV
Cranial nerve VI
Occipital lobe

11 June 2023 3rd real exam

1) A 25-year-old woman is brought to the Emergency Department following a road traffic car acci-
dent. On arrival her blood pressure is 110/50mmHg, heart rate 92/min and respiratory rate 28/
min. On examination there is decreased air entry on the right side as well as dullness on percus-
sion and fracture of 2 ribs. Which one of the following is immediate treatment of the patient’s
condition?
analgesia-sedatives
CPAP
endotracheal intubation and ventilation
insertion of Chest drain underwater seal
Strapping the complete chest

2) A 26-year-old Para 1+2 came to the emergency department 3 weeks after an abortion, with severe
vaginal bleeding. She reported undergoing surgical evacuation 3 weeks earlier, but did not re-
turn to receive the pathology report of the specimen. Her beta-subunit titre was above 100,000
milli-international units/milliliter, her ultrasound showed an intra-uterine mass 4x4 centimeters.
What is the most appropriate management?
Resuscitation followed by conservative management
Resuscitation and uterotonics
Resuscitation and re-evacuation
Start chemotherapy.
Hysterectomy

67
3) Three days after undergoing uncomplicated caesarean delivery of a boy with weight 4 kg at term,
a 38-year-old, gravida 3, para 3 woman has sudden onset of right-sided chest pain and cough
productive of bloody sputum. Pregnancy was complicated by gestational diabetes. She smoked
and pack of cigarettes daily for 10 years but quit when she became pregnant. Her temperature
is 37PC (98.8°F), pulse is 108/min and regular, respiratory rate is 28/min, and blood pressure is
116/70 mm Hg. Pulse oximetry on room air shows an oxygen saturation of 92%. Physical exam-
ination shows mild pallor of the skin and conjunctivae, Auscultation of the lungs discloses de-
creased air entry over the right lung base: a pleural rub is heard. There is no calf pain or adema
of the lower extremities ECG shows no abnormalities X-ray of the chest shows a wedge-shaped
opacity over the right lower lung zone with obliterated the costophrenic angle. Which of the fol-
lowing is the most likely diagnosis?
acute bronchitis
chronic obstructive pulmonary disease
pleural effusion
Post operative pneumonia
Pulmonary embolism

4) You are conducting a clinic visit with a 54-year-old man with hypertension. He made the appoint-
ment earlier this morning because he began feeling short of breath. As you enter the room, you
note that he is sitting forward and breathing heavily, Given this scenario, what interviewing tech-
nique is the most patient-centered?
se open-ended questions to allow the patient to tell his story.
start by obtaining a list of all issues that the patient wants to discuss during this visit
direct the interview using closed questions related to his reason for this visit
elicit the broader personal/psychosocial context of the symptoms
ask the patient about his beats regarding faster versus Western treatment options

5) A 4-year-old child (4 years) presents to the Emergency Department with fever 400C, headache and
two attacks of convulsions over the past two hours. Examination shows he is irritable and has
neck rigidity. His mother states he had fever 380C and mild nasal discharge. What is the most
appropriate investigation done?
blood picture
blood culture
CT head
Lumbar puncture
Serum electrolytes

6) A 61-year-old man comes to the office because of a six-month history of urinary hesitancy, weak
urinary stream, and nocturia. He has voided two to three times nightly during this time. He has
no history of urinary tract infections, serious illness and takes no medications. Vital signs are
within normal limits. Digital Rectal examination discloses a moderately symmetrically enlarged
prostate with soft consistency and no tenderness on palpation. Total Serum prostate-specific
antigen (PSA) concentration is 8.9 ng/ml [N<4.0 ng/mL] 3 week later, repeat measurement of
serum PSA concentration is 8.8 ng/mL Abdominal ultrasound revealed a Prostate size of 60 gm,
post- micturition Residual urine of 70 ml. Which of the following is the most appropriate next
step in management?
Finasteride 5 mg/day/ orally

68
Tamsulosin 0.4 mg/day / orally
transrectal ultrasound-guided biopsy
transurethral biopsy of the prostate
transurethral resection of the prostate

7) A 35-year-old Para 3 has painless vaginal bleeding at 37 weeks’ gestation. Which of the following
measures is immediately indicated?
Cesarean section.
b- Induction of labor.
Coagulation profile.
Rupture of membranes
Fatal monitoring

8) A 22-year-old gravida 3 Para 0+2, sure of her dates, came at 9 weeks gestation with vaginal spot-
ting An ultrasound scan has been performed on weekly basis since the diagnosis of her pregnan-
cy. They all failed to detect a fetal pole in spite of a gestational sac diameter matching her dates.
What is the appropriate management?
Parenteral progesterone.
Cervical cerclage.
Folic acid.
Termination of pregnancy,
Repeat ultrasound after one week

9) A 65-year-old woman with no other significant past medical history presents with large mass in
the right breast. The mass measures approximately 6 cm in diameter and appears to be fixed to
the chest wall. The mass is being progressively enlarging. In addition, bulky right axillary lymph
nodes are palpable. Following mammography, what should be the next step in this patient’s eval-
uation?
Fine Needle Aspiration Cytology (FNAC)
Core biopsy
Excision biopsy
Incision biopsy
-Breast ultrasonography

10) A three-year-old child is referred to the child psychiatrist with poor social interaction with his
peers and delayed verbal communication. The mother also noticed repetitive stereotypical be-
havior. Which is the most reliable method to confirm the cause of referral?
CT brain
developmental assessment
hearing test
M-CHAT (modified checklist for Autism)
MMR antibodies

11) A 31-year-old woman presents with right breast throbbing pain and swelling of 2 days duration,
69
with fever and chills. She is currently breast-feeding Her temperature is 38.8 C Pulse 92/m BP
130/75. The right breast has an area of erythema and edema that is tender. Which one of the
following is the most likely clinical diagnosis?
breast abscess
breast cyst
breast inflammatory carcinoma
fibro adenosis
fibroadenoma

12) A 48-year-old man, a heavy smoker, has chronic arthritis which makes him receiving NSAID (Non-
steroidal Anti-inflammatory) regularly. Has been admitted with recent sudden onset of very se-
vere pain in the epigastrium radiating to the back and right shoulder tip of 3 hours duration. His
pulse is 88 beats per minute and his blood pressure is 120/70 mmHg; his abdomen has board-
like rigidity and does not move with respiration, which is mainly thoracic. What is the appropriate
diagnosis?
Acute cholecystitis
Gastritis.
Leiomyoma of stomach.
Perforated duodenal ulcer.
Periampullary carcinoma.

13) A 6-year-old boy is brought into the emergency room by his mother for walking with a limp for
several weeks. On examination, the patient has tenderness over his right thigh without evidence
of external trauma. An x-ray of the pelvis shows a right femoral head that is small and denser
than normal. Which of the following is the most likely diagnosis?
Blount disease
Dysplasia of the hip
Legg-Calve-Perthes (LCP) disease
Slipped capital femoral epiphysis (SCFE)
Talipesequinovarus

14) An 11 months old boy is brought to the Emergency Department with 6 episodes of diarrhea and 4
episodes of vomiting daily for the past 2 days. On examination, he has a dry mouth, sunken eyes,
and depressed fontanelle. What is the most important initial management steps?
give antidiarrheal medication
give parenteral antiemetic
insert IV line and correct dehydration
insert nasogastric tube for feeding
send home advising the mother for giving excess fluids

15) A mother of an 8 months old child decided to give him cow’s milk instead of breast milk. He start-
ed to develop rash on his face only with no any other symptoms or signs. What is the most likely
diagnosis?
atopic dermatitis
bacterial infection
70
fungal infection
slapped checks
urticarial wheel

16) A 26-year-old Paral had a missed period for 8 days 2 months after IUCD removal. Serum B-hCG
level was 550 u/L, raised to be 875 u/L after 48 hours with no evidence of intra-uterine sac by
transvaginal ultrasound. What is the most likely diagnosis?
Too early intra-uterine pregnancy.
Extra-uterine pregnancy.
Threatening fetal demise.
No intra or extra uterine pregnancy.
Intra-uterine missed abortion.

17) A 43-year male fell off a horse in a farm. He cannot remember the last tetanus vaccination. On
examination he was well apart from a 10 cm deep, lacerated, dirty wound in his left leg. What are
the important initial steps in the treatment?
Booster dose of tetanus vaccine, tetanus immunoglobulin, antibiotics and cleaning the wound
Booster dose of tetanus vaccine, immunoglobulin, antibiotics and primary closure of wound.
Booster dose of Tetanus vaccine, antibiotics and cleaning the wound.
Tetanus immunoglobulin, antibiotics and cleaning the wound.
Tetanus immunoglobulin, antibiotics and primary wound closure.

18) A 25-year-old female kindergarten teacher comes to your office for evaluation of a cough she has
had for one week. The preceding week she had symptoms of sore throat, malaise, and low-grade
fever. She reports that episodes of coughing are so severe that vomiting is induced. She was
evaluated at a walk-in clinic was diagnosed with bronchitis. Treatment with hydrocodone cough
syrup and amoxicillin has not helped. On examination, her lungs are clear. A chest radiograph &
CT showed infiltrations. Her laboratory results reveal lymphocytopenia. What is the most likely
diagnosis?
Avian Flu
common cold
COVID 19
seasonal Influenza
Swine flu

19) A ten-year-old boy presents to the outpatient clinic with recurrent attacks of headache over the
last week. Headache is unilateral, pulsating, associated with intense vomiting. Which is the most
appropriate management step?
CSF examination
ENT examination
Fundus examination
MRI brain
X-ray skull

20) A 57-year-old woman presents to the emergency department with cramping/colicky abdomi-
71
nal pain. The current episode of pain began12 hours ago, following a heavy fatty meal. The pain
began slowly and rose in intensity to a plateau over the course of several hours. On clinical exam-
ination, she catches her breath while palpation in the right hypochondrium. She says her pain is
radiating to the inferior angle of right scapula. Which of the following is the most likely diagno-
sis?
appendicitis
cholecystitis
colitis
gastritis
pancreatitis

21) A 65-year-old man presents to ER with fever, rigors and upper abdominal pain for one day, not
improving with analgesics. On examination, his temp 38°C, pulse 120/min and BP 130/85mmHg.
He is jaundiced. Lab. tests were WBC 21,000/L, AST 68 IU/L, ALP 442 IU/L, bilirubin 4.3 mg/L.
What is the most likely diagnosis?
Acute appendicitis
Acute cholecystitis
Acute cholangitis
Acute intestinal obstruction
Acute hepatitis

22) A 52-year-old woman comes to the office because of a 2-day history of pain in her right call when
walking that occurs in the evening. Twelve hours later, she had onset of rapid swelling of her
right leg and she now has difficulty in walking. She says her symptoms began after she returned
from a long vacation flying for 10 hours: She has a 10-year history of type 2 diabetes mellitus
treated with metformin. Her temperature is 37°C, pulse is 75/min, respirations are 16/min, and
blood pressure is 120/70 mm Hg. Physical examination shows edema and tenderness (pain) to
palpation of the right calf. No erythema is noted. Pedal pulses are normal. Which of the following
is the most likely diagnosis?
arterial embolism
cellulitis
deep venous thrombosis
diabetic foot infection
myositis

23) A ten months old boy suddenly develops fever 40.1 C and is rushed by his parents to the Emer-
gency Room. As soon as he arrives he starts to convulse with tonic clonic fits. Respiratory rate
40/min, Heart rate 110/ min Which is the most appropriate initial management step?
ensure patent airway and IV diazepam
IV antibiotics and IV diazepam
IV fluids and IV diazepam
paracetamol suppositories
rectal diazepam

24) A 14-year-old male patient presented with fracture tibia while doing sand boarding in Fayoum.
There is a controversy among surgeons regarding how to treat his fracture. One prefers open
72
reduction while the other advised closed reduction What is the disadvantage of open reduction in
this patient?
Decreases trauma to the fracture site
Produces a higher incidence of nonunion
Produces a shorter healing time
Reduces the risk of infection
Requires longer periods of immobilization

25) A 50-year-old man is brought to the emergency department by paramedics 2 hours after he vom-
ited a large amount of blood. He has no history of similar episodes. One year ago, he sustained
trauma to his back L2 Stable fracture. He takes nonsteroidal anti-inflammatory drugs as needed
for pain. He has no other history of serious illness and takes no routine medications He drinks 3
glasses of wine daily. His temperature is 37°C, pulse is 115/min, respirations are 19/min, and
blood pressure is 100/60 mm Hg. Physical examination shows mild conjunctival icterus. Cardio-
pulmonary examination discloses no abnormalities. Abdominal examination discloses mild asci-
tes. The liver edge is palpated 3 cm below the right costal margin. The spleen tip is palpated 4 cm
below the left costal margin There is no edema of the lower extremities. Nasogastric aspiration
yields 250 mL of bright red blood. The patient is resuscitated. 4units of packed red blood cells are
transfused, and 2 liters of 0.9% saline is administered. Which of the following is the most likely
diagnosis?
esophageal carcinoma
esophageal varices
esophagitis
gastric carcinoma
peptic ulcer

26) Patient referred to emergency room after road traffic accident, on examination there is disturbed
conscious level, irregular breathing, fracture both bones in left leg blood pressure was 100/70.
What is the first priority in treating this patient?
Blood transfusion
Ensure air way & fixneck
Fixation of limb fracture
Normal saline infusion
Ultrasound abdomen & pelvis

27) A 33-year-old Para 3 comes to the physician for an annual examination. She has no complaints.
Obstetric history is significant for 3 normal vaginal deliveries with gestational diabetes during
the last 2 pregnancies. She doesn’t take any medications. Family history is significant for pater-
nal coronary artery disease. Physical examination is unremarkable. Which one of the following
investigations should this patient perform?
Chest X-ray every 3 years.
Coronary angiography every 3 years.
Fasting glucose testing every 6 months.
Mammography every 3 years.
Pelvic ultrasound every 3 years.

73
28) A 37-year-old, Para 1+2, came for antenatal care at 10 weeks gestation. Her first 4.5 kg baby
was delivered vaginally after 2 abortions. She has no history of gestational diabetes but a posi-
tive family history. Her body mass index is 27 kg/m2. What puts her more under a risk to develop
gestational diabetes?
Her previous macrosomic baby.
Her previous two abortions.
Her body mass index.
Her family history for diabetes.
Her age in the current pregnancy

29) A 75-year-old man comes to the office because of a 3-day history of cough productive of yel-
low sputum, stabbing chest pain, and fever. He has hypertension treated with ACE inhibitors He
has smoked one pack of cigarettes daily for years. His temperature is 39.0°C, pulse is 100/min,
respirations are 28/min, and blood pressure is 140/90 mm Hg. Pulse oximetry on room air shows
an oxygen saturation of 89%, Physical examination shows the patient was sweating No cyanosis
is noted. Auscultation of the lungs discloses decreased air entry, wheezes, and crackles over the
left hemithorax and dullness to percussion on the left side of the chest Supplemental oxygen
therapy is begun. Which of the following is the most appropriate next step in management?
bronchoscopy
intravenous antibiotic therapy
oral antibiotics
start antibiotic after results
thoracentesis

30) Before abdominal closure at the end of open splenectomy, the scrub nurse informed you of a
missing abdominal towel. Recounting, thorough search in the room all failed to find the towel.
You could not find the towel intra-abdominal and asked for intra-operative X-ray which did not
show the radio-opaque marker thread. What is the proper action?
Close the abdomen and do not notify the incident.
Close the abdomen and notify the legal authorities.
Call one of the relatives outside the OR, and notify him before closure
Notify the OR head nurse, keep a copy of the X-ray, close with incident report.
Call the hospital manager before closure and notify the patient postoperative.

31) A 4 week old boy has repeated attacks of vomiting since birth that occur soon after feeding.
irrespective of the amount of milk he takes. He is otherwise well. Birth weight was 3,5 kg and
on examination his weight is 4.4kg and no other abnormality is detected. What is the most likely
diagnosis?
cows milk protein allergy
gastro-oesophageal reflux
lactose intolerance
pyloric stenosis
urinary tract infection

32) A 70-year-old women is seen in the outpatient clinic for left iliac fossa abdominal pain of 6
days duration. On examination; Oral temperature is 39.5° C, Pulse: 110 b/min, blood pressure:
74
140/85. Abdominal examination revealed localized tenderness and rigidity in the left iliac fossa.
Hemoglobin: 12.5 gm/dl, White cell count: 18,000 /cubic mm. What is the most predicted provi-
sional diagnosis?
Pseudomembranous colitis
Acute Gastroenteritis.
Diverticular peri-colic abscess.
Irritable bowel syndrome.
Cecal diverticulitis.

33) A severely traumatized woman is seen in the emergency room with a Glasgow coma scale of 5T.
Her family asks what the significance of this measurement is. Which of the following statements
you will say it to the family regarding the Glasgow coma scales of this patient?
A high score correlates with a high mortality rate.
It includes measurement of intracranial pressure (ICP).
It includes measurement of pupillary reflexes.
It includes measurement of verbal response, motor response and eye opening.
It serves to assess the long-term sequelae of head trauma.

34) A 22 month-old girl was fussy, crying all the time and could not sleep for the past two days. She
kept on pulling her left ear repeatedly. She suddenly stopped crying and her parent’s noticed pus
coming out of her ear. Temperature is 39.2°C, Respiratory rate 26/min, Heart rate 100/min. The
pediatrician explains that she has to receive an antibiotic. What is the most appropriate antibiotic
to prescribe?
Amoxicillin
Ciprofloxacin
Clindamycin
Erythromycin
Flucloxacillin

35) A 65-year-old man presents to ER with fever, rigors and upper abdominal pain for one day, not
improving with analgesics. On examination, his temp 38°C, pulse 120/min and BP 130/85mmHg
He is jaundiced. Lab. tests were WBC 21,000/L, AST 68 IU/L, ALP 442 IU/L, bilirubin 4.3 mg/L
What is the most likely diagnosis?
Post phlebitic syndrome
Cellulitis
Arteriovenous fistula
Hypoproteinemia
Lymphatic obstruction

36) A 32-year-old Para 2, presented to the emergency department shocked with lower abdominal
pain and mild vaginal bleeding. Her pregnancy test was positive and ultrasound showed an emp-
ty uterus and hemoperitoneum. She was diagnosed as acutely disturbed ectopic pregnancy. She
underwent a laparotomy and left salpingectomy followed by cervical dilatation and endometrial
curettage. Which one of the following pathologies of her curettage would be most likely?
Chorionic tissue.

75
Arias Stella reaction.
Simple endometrial hyperplasia.
Complex hyperplasia.
Atrophic endometrium.

37) A 75-year-old woman comes to the emergency department because of a 8-hour history of left-
sided abdominal pain. This morning 2 hours ago,she passed stool with dark-colored blood clots.
She has type 2? diabetes mellitus and hypertension treated with ACE inhibitors. She has no histo-
ry of gastrointestinal illness. Her temperature is 38.1°C (100.6°F), pulse is 120/min, respirations
are 18/min, and blood pressure is 110/85 mm Hg. Abdominal examination discloses tenderness
(pain) to palpation along the descending colon. Laboratory studies show: CT scan of the abdomen
with contrast shows no free air. Which of the following is most likely to confirm the suspected
diagnosis?
abdominal ultrasound.
barium enema
colonoscopy
CT scan of the abdomen with contrast
MRI of the abdomen.

38) A 22-year-old woman who underwent cesarean delivery two days ago, has persistent fever of
38.9°C, despite antibiotic therapy. The urinalysis, wound, breasts, and uterine fundus are normal
on examination. A CT scan of the pelvis is suggestive of septic pelvic thrombophlebitis. Which
one of the following is the best therapy for this condition?
Continued triple antibiotic therapy.
Discontinue antibiotic therapy and initiate IV heparin.
Continue antibiotic therapy & begin IV heparin.
Surgical embolectomy.
Streptokinase therapy.

39) A 37-year-old woman with right lower extremity edema is evaluated because of the sudden on-
set of shortness of breath and pleuritic chest pain. A diagnosis of pulmonary embolism is made.
Which of the following signs, if present on physical examination, would be the most specific
indicator of pulmonary arterial hypertension in this patient?
increased jugular venous pressure
P2 louder than A2
peripheral edema
presence of an S3
pulmonary crackles

40) A 34-year-old primigravida, at 28 weeks, presented at her routine antenatal visit with a com-
plaint of vaginal discharge diagnosed to be vaginal candidiasis, for which she received two suc-
cessive courses of local treatment with incomplete resolution. Her urine analysis showed pus
cells 50- 60/high power field, sugar 1+. On ultrasound examination, the amniotic fluid index was
23. What is the next step of management?
Schedule for the next prenatal visit after 2 weeks
Repeat local treatment for infection using another preparation.
76
Request a glucose tolerance test.
Systemic antifungal treatment
Termination of pregnancy

41) A woman presented 5 days after delivery, complaining of a painful swollen left leg. She is found
to have leukocytosis and low grade fever. What is the most appropriate immediate management
before diagnosis of DVT is confirmed?
Warfarin oral treatment.
Prophylactic heparin.
Therapeutic heparin.
Low dose aspirin tablets.
Diosmin (venotonic) tablets,

42) A 50-year-old man is brought to the emergency department by paramedics 2 hours after he vom-
ited a large amount of blood. He has no history of similar episodes. One year ago, he sustained
trauma to his back L2 Stable fracture. He takes non-steroidal anti-inflammatory drugs as needed
for pain. He has no other history of serious illness and takes no routine medications. He drinks
3 glasses of wine daily. His temperature is 37°C, pulse is 115/min, respirations are 19/min, and
blood pressure is 100/60 mm Hg. Physical examination shows mild conjunctival icterus. Cardio-
pulmonary examination discloses no abnormalities. Abdominal examination discloses mild asci-
tes. The liver edge is palpated 3 cm below the right costal margin. The spleen tip is palpated 4 cm
below the left costal margin. There is no edema of the lower extremities. Nasogastric aspiration
yields 250 mL of bright red blood. The patient is 4 units of packed red blood cells are transfused,
and 2 liters of 0.9% saline is administered. Which of the following is the most appropriate next
step in management?
balloon tamponade
endoscopic band ligation
placement of a selective portosystemic shunt
placement of a splenorenal shunt
placement of a transjugular portosystemic shunt

43) A mother of a five-year-old girl comes to the pediatrician upon the referral from the General
Practitioner, who performed her preschool assessment and explained that she has to change her
feeding habits as she refuses all food except potato chips and biscuits. Which is the most likely
screening tool the General Practitioner used for assessment?
Bmiforage
iengthforage
skinfoidthickness
weightforage
weightforheight

44) A 30-year-old, PG pregnant at 36 weeks started to develop non-proteinuric mild hypertension


(140/90 mmHg) and mild pitting edema of lower limbs. How to proceed for proper management?
Start alpha methyldopa orally.
Follow up with rest and low-salt diet.
Termination of pregnancy.
77
Magnesium sulfate injection.
Low molecular weight heparin

45) A previously healthy, obese 26-year-old G 1 PO has a positive glucose challenge test at 28 weeks
gestational age (GA). Follow-up testing confirms the diagnosis of gestational diabetes mellitus
(GDM). The patient asks what this diagnosis will mean for her baby. Which of the following can
be a significant consequence of poorly controlled GDM to the fetus?
Hyperglycemia
Macrosomia
Cataracts
Cerebral edema
Cephalohematoma

46) An 18-years old healthy woman came to your clinic with a recent purified protein derivatives
tuberculin skin test that is positive after routine examination for employment. Clinical examina-
tion was irrelevant with no signs of disease. Chest examination was free. A chest radiograph was
done and is negative. What is the most appropriate treatment?
follow-up after 3 months
INH and rifampin for 6 months
Isoniazid (INH) for 6 months
INH, rifampin and streptomycin for 12 months
Streptomycin for 6 months

47) A 30-year-old, G5, Para 0+4 was diagnosed to have antiphospholipid syndrome. She is on low
molecular weight heparin till 30 weeks gestation. She starts to have mild hypertension, protein-
uria (+) with normal kidney functions but liver enzymes in the high normal range. What is the
proper plan for management?
Immediate termination of pregnancy.
Termination of pregnancy at 34 weeks.
Termination of pregnancy at 36 weeks.
Termination of pregnancy at 38 weeks.
-Termination of pregnancy at term.

48) A five year old boy presents to the outpatient clinic with generalized edema which is more ap-
parent in the morning. Respiratory rate 18/min, Heart rate 90/min, BP 80/45. Laboratory inves-
tigations show serum albumin=1.4 g/dL (3.5-5.5 gm/dl), cholesterol = 360 mg/dL (150-199 mg/
dl), urinalysis revealed +++ proteins and RBC 1-2/HPF. Serum creatinine is normal. The physician
requests a kidney biopsy. What is the most probable pathological diagnoses?
Focal segmental glomulosclerosis
IgA nephropathy
Microangiopathy
Membranous glomerulopathy
Minimal change glomerulonephritis

49) A 72-year-old man presents with pain in his right big toe. The pain started this morning and is
78
getting progressively worse, and he is unable to bear weight on that foot. He is febrile, has a
pulse of 90/min, and has a blood pressure of 136/86 mm Hg. The first metatarsophalangeal
joint of the right foot is swollen, warm, and erythematous. It is tender to palpation and there is
decreased movement. Which one of the following is the best agent for the acute management of
this patient’s condition?
intravenous colchicine
oral acetaminophen
oral allopurinol
oral indomethacin
oral probenecid

50) A 52-year-old woman comes to the emergency department because of a 3-week history of short-
ness of breath and nonproductive cough. The shortness of breath has progressively worsened
during the past 2 days. She also has a 3-week history of low back mild pain. Ibuprofen has pro-
vided minimal relief of her pain. She has no history of serious illness and takes no routine medi-
cations. She began smoking at age 32 and has smoked one pack of cigarettes daily. Her tempera-
ture is 37.0°C, pulse is 105/min, respirations are 26/min, and blood pressure is 180/118 mm
Hg. Pulse oximetry on room air shows an oxygen saturation of 88 %. Physical examination shows
jugular venous distention to 3cm above the sternal angle. Auscultation of the lungs discloses
bilateral wheezing and basilar crackles. Cardiac examination discloses an S3. There is bilateral
edema of the lower extremities. Chest x-ray shows cardiomegaly, bilateral infiltrates, and bilat-
eral pleural effusions at the lung base. Which of the following is the most likely diagnosis?
asthma
chronic obstructive pulmonary disease
myocarditis
pneumonia
right-sided heart failure

51) An 18-year-old football player is seen in the emergency ward with severe knee pain incurred
after being hit by a tackler while running. Physical examination is consistent with tearing of a
meniscus. Which movement causes this injury?
Compression
Femoral condylar fracture
Flexion and rotation
Hyperextension
Simple hyperflexion

52) A 66-year-old woman is admitted to the hospital because of a 3-month history of shortness of
breath that has progressively worsened during the past month. Initially the symptom occurred
with moderate exertion; now she has shortness of breath at rest. She also has a 1-week history
of swelling of her legs and feet. She has hypertension treated with 5 mg bisoprololy. Her tem-
perature is 37 °C), pulse is 120/min, respirations are 28/min; and blood pressure is 165/95 mm
Hg in both extremities. Pulse oximetry on room air shows an oxygen saturation of 92%. Physical
examination shows no cyanosis. Jugular venous distention is present 4 cm above the sternal an-
gle. Auscultation of the lungs discloses bilateral crackles in the lower and midlung fields. Cardiac
examination discloses an S3 and S4. There is bilateral pitting edema of the lower extremities to
the knee. An ECG shows sinus tachycardia and bilateral and biventricular hypertrophy. Which of
the following is the most appropriate pharmacotherapy?
79
Dopamine, furosemide,
Epinephrine (adrenaline),, and labetalol
Furosemide and nitroglycerin
Furosemide and verapamil
Nitroglycerin and Verapamil

53) A 65-year-old female patient is admitted to the emergency department with a short history of
fresh blood per rectum, and long history of altered bowel habits. The patient is pale, blood pres-
sure: 140/85, Pulse: 75/min. There are residual stains of blood on rectal examination. Her bowel
is opened with no new attacks of bleeding. What is the next immediate step for management?
Abdominal ultrasonography.
Oesophago-gastro-duodenoscopy.
Mesenteric CT angiography.
Colonoscopy.
Abdominal plain X

54) A 6-year-old boy with Down syndrome is brought by his mother to the clinic feeling worried that
he had fever for the last three weeks that ranged from 37.8° C and 38.3 °C associated with an-
orexia and loss of weight. On examination he looked pale. His was two kg less than his weight in
his last checkup that was four months before. What is the most appropriate initial investigation?
bone marrow aspiration
blood culture
c-reactive protein
complete blood picture
Epstein Barr virus antibodies

55) A 32-year-old Para 2, both deliveries were by CS. Ultrasound done at 32 weeks shows that the
placenta is grade 2 and located anteriorly down to the level of internal os. Which one of the fol-
lowing complications is the most likely risk?
Placenta accreta
Abruption placenta

56) A 60-year-old woman comes to the emergency department 2 hours after she vomited coffee
ground-like material. She has no history of similar symptoms. She has osteoarthritis of both hips
and hypertension. She is on daily non-steroidal, anti-inflammatory tablet and Enalapril tablet.
She started smoking at age 30 and has smoked one pack of cigarettes daily She drinks one glass-
es of wine once a week. She is 160 cm tall and weighs 85 kg. Her temperature is 36.9°C, pulse is
110/min, respirations are 20/min, and blood pressure is 120/70 mm Hg. Physical examination
shows no other abnormalities. Nasogastric aspiration yields 200 mL of coffee-ground material.
A 0.9% saline stomach lavage cleared the aspirate Which of the following is the most appropriate
next step in diagnosis?
angiography of the celiac axis
CT scan of the abdomen
esophageal Manometry
ultrasonography of the abdomen

80
upper gastrointestinal endoscopy

57) A 25-yar-old PG came for her first antenatal visit at 6 weeks gestation. She was clinically free
with satisfactory laboratory tests. What would be the recommended ROUTINE supplement(s) as
in all pregnancies?
Iron supplements only.
Daily calcium tablets.
Iron and folic acid.
Folic acid and vitamin D.
Pregnancy multivitamin formula.

58) room with a history of 2 eclamptic fits at home. Examination revealed blood pressure 180/110,
marked edema, heavy proteinuria. No fetal cardiac pulsations could be detected by ultrasound
scan. What is the appropriate management?
Immediate termination of pregnancy.
Stabilization of her condition and then termination after 1-2 hours.
Anticonvulsant and antihypertensive drugs for 24hours.
Conserve until spontaneous expulsion of the dead fetus.
No need to control fits or hypertension as the baby is dea

59) A 40 year old mother gave birth at home to a child with Down Syndrome. She visits the GP who
explains that there seems to be no associated abnormality but that the baby has to be screened
for any hidden anomaly. Which investigations is most likely requested first to exclude any hidden
anomaly?
barium meal
chest x ray
echocardiography
head ultrasound
renal ultrasound

60) Three hours after difficult thyroidectomy, for huge retrosternal goiter, A 30-year-old woman de-
veloped agitation and difficulty in breathing, tachycardia. Anterior cervical swelling was detected
in front of the wound with bluish discoloration of the overlying skin but the overlying dressing
was dry. What is the appropriate immediate step
Estimation of serum calcium level.
High flow 02 mask.
Insertion of on and tracheal tube.
Reopening of the cervical wound.
Slow injection of calcium gluconate

61) Over the past 6 months, a 60 years old woman with long standing duodenal ulcer disease has
been complaining of anorexia, nausea, weight loss and repeated vomiting. She recognizes un-
digested food in the vomitus. Examination and workup reveal: Sodium: 130 mEq/L, Potassium:
2.9 mEq/L, Chloride: 89 mEq/L, Bicarbonate:19 mEq/L, PH: 7.64, BE +3.5. What is the most likely
diagnosis?
Carcinoma of the fundus of the stomach.
81
Penetrating ulcer.
Pyloric obstruction due to cicatricial stenosis of the antrum.
Reflux esophagitis.
Anorexia nervosa.

62) A 32-year-old, Para4 committed an illegal abortion at 8 weeks gestation by an unqualified doctor
at his clinic. One week later she presented with high grade fever, moderate vaginal bleeding with
offensive odor. What is the most likely diagnosis?
Incomplete evacuation of conceptus.
Incomplete septic abortion.
Undiagnosed extra-uterine pregnancy.
Trichormonas vaginalis infection.
Severe urinary tract infection.

63) An 8 year old girl has asthma, and is taking low-dose inhaled corticosteroids twice daily and a
short- acting bronchodilator as required. She is concordant with her treatment and her inhaler
technique is good. For three months she has had frequent cough at night and mild exercise-in-
duced wheeze. What is the most appropriate next step in her treatment?
add leukotriene receptor antagonist
add long-acting bronchodilator
add oral modified release theophylline
add short course of oral corticosteroid
increase dose of inhaled-corticosteroids

64) A 2 day old full term baby is admitted to the NICU because of poor feeding and green-stained
vomitus. Birth weight is 3.5kg. On examination he is irritable, and abdomen is mildly distended.
Temperature is 370C, respiratory rate is 40/minute, heart rate 120/minute. No other abnormality
detected. Which is the most likely investigations to confirm the diagnosis?
endoscopy.
lower GIT contrast study.
plain abdominal radiograph.
ultrasound of the abdomen.
upper GIT contrast study

65) A 75-year-old man is found by his internist to have sudden dizziness, visual disturbances, numb-
ness in one limb side. His blood pressure is 170/100 with a carotid bruit. Which of the following
is the best initial diagnostic investigation?
Doppler ultrasonography (duplex).
Spiral CT angiography.
Arch aortogram with selective carotid artery injections.
Magnetic resonance arteriogram (MRA).
Cervical spine X ray.

66) A 44-year-old woman comes to the emergency department because of a 1-day history of severe
82
pain in her upper abdomen that radiates to her back, nausea, and vomiting. She has had 3 epi-
sodes of emesis during this time. She has no history of serious illness and takes no medications.
She does not smoke cigarettes. Her temperature is 37.0°C (98.6°F), pulse is 11/min, respirations
are 22/min and shallow, and blood pressure is 118/72 mm Hg. Pulse oximetry on room air shows
an oxygen saturation of 97%? The abdomen is soft and mildly distended. There is sever right
upper quadrant and epigastric tenderness (pain) to palpation. No organomegaly is noted. Labora-
tory studies Which of the following is the most likely diagnosis?
cholangitis
empyema of the gallbladder
pancreatic pseudocyst
pancreatitis
perforated peptic ulcer

67) A 65-year-old male, with uncontrolled diabetes, presents to the emergency department 36 hours
after drainage of a perianal abscess, with exacerbating genital pain. Examination reveals a tem-
perature of 38 C and a pulse rate of 120 b/m. His scrotum is edematous, red, and tender, and its
skin appears dusky, with subcutaneous crepitus on palpation. What is the most likely diagnosis?
Cellulitis.
Fournier’s gangrene.
Epididymo-orchitis.
Isolated scrotal abscess.
Erysipelas.

68) A 3 month old infant is found to have a continuous murmur all over the precordium during her
routine well baby check up. The infant is 6 kg in weigh and appears well. All peripheral pulses are
present and easily palpable. Oxygen saturation is 96%. What is the most likely finding on echo-
cardiography?
atrial septal defect
coarctation of the aorta
dextrocardia with situs inversus
persistent ductus arteriosus
ventricular septal defect

69) A 32-year-old Para 2+1 had her 20 weeks anomaly scan. A placenta previa extending over the
internal cervical os has been identified. What is the appropriate management?
Another trans-abdominal scan at 32 weeks.
To be re-evaluated at time of term delivery.
Re-assessment at 38 weeks:
Trans-vaginal cervical cerclage.
Placental site identification every two weeks.

70) A 42-year-old, Para4 pregnant at 38+4 weeks consulted her doctor about recurrent, painless
mild attacks of vaginal bleeding during last week. She is clinically free but ultrasound scan
showed a posterior placenta previa marginalis. What is the appropriate management?
Immediate watchful induction of labour.

83
b- Wait for spontaneous onset of labour.
Cesarean section is warranted.
d- Start of tocolytic drugs.
Counsel for cesarean hysterectomy.

71) When a professional secret may be disclosed?


if the court asks for it
if the patient died
if the patient is in coma
if the patient’s brother asked for it
if the physician is accused by patient’s family

72) A 27-year-old G IPO at 15 week’s gestation has a history of chronic 3(36.5°C) hypertension. She
presents for her first prenatal visit. Vital signs: T 98°F P 66 beats/min, R 16 breaths/min, BP
146/98 mmHg. She stopped taking her BP medication 6 weeks ago when she found out that she
was pregnant. Of the following medications, which is the best choice for treating chronic hyper-
tension in pregnancy?
Hydralazine
Hydrochlorothiazide
Labetalol
Lisinopril
Atenolol

73) A 22-year-old male suffered sudden pain which started at central abdomen then became local-
ized in the lower right side of the abdomen. Examination showed right iliac fossa rebound ten-
derness. Which of the following is a feature of non-complicated appendicitis?
Temperature 39.5
WBCs 19000
Collection in right iliac fossa and pelvis in U/S
Generalized abdominal rigidity
Crossed tenderness on pressing left iliac fossa

74) A 52-year-old man comes to the office because of a 2-day history of sore throat, cough pro-
ductive of a small amount of whitish sputum, nasal discharge, and body aches. He has type 2
diabetes. mellitus on metformin and hypertension treated with ACE inhibitors. He smokes occa-
sionally. His temperature is 38.0°C, pulse is 85/min, respirations are 22/min, and blood pressure
is 140/90 mm Hg. Pulse oximetry on room air shows an oxygen saturation of 98%. Physical exam-
ination shows no cyanosis. The throat is mildly congested. Auscultation of the lungs discloses
scattered wheezes. Air entry is equal bilaterally. Heart sounds are normal. Which of the following
is the most likely diagnosis?
asthma
bronchiolitis
bronchitis
laryngitis
pneumonia
84
75) A healthy full term baby boy is born to mother who was confirmed to have acute hepatitis B
infection during this pregnancy. Which is the most appropriate preventative intervention to the
baby?
give the full course of hepatitis B vaccine
give the full course of hepatitis B vaccine and hepatitis B immunoglobulin
give hepatitis B immunoglobulin only
give a single dose of hepatitis B vaccine
test for Hepatitis B surface antigen and decide intervention according to result

76) A 42-year-old male presents with a complaint of one day inability to close his left eye and devi-
ation of the angle of the mouth while speaking. The patient is diabetic on metformin. On exam-
ination: Temperature 370 C, Pulse 75/m, BP 140/85, chest, and heart no abnormality detected.
Patient is unable to wrinkle forehead on the left side, unable to close the left eye and unable to
blow the cheek Angle of the mouth deviates to right on attempt to show the teeth. There is no
sensory deficit over the face. Examination of the tongue shows no abnormality. What is the neu-
rological diagnosis in this patient?
ischemic stroke
left facial nerve herpes zoster
left sided bell’s palsy
left sided cerebellopontine angle lesion
left sided trigeminal nerve palsy

77) An eight-year-old girl develops fever 39° C and headache for two days. She receives paracetamol
suppositories. This morning the headache markedly increases in severity and she develops pro-
jectile vomiting and convulsions. On examination conscious level was disturbed together with
neck rigidity and photophobia. What is the most probable diagnosis?
idiopathic epilepsy
meningitis
migraine
paracetamol toxicity
posterior fossa tumor

78) A 3-month-old infant is brought to the clinic by his mother with clear nasal discharge and mild
cough. The general practitioner explains he has a mild viral infection. Two days later the infant
presents to the Emergency Room with cough, difficult breathing and wheezes. Temp was 38° C,
Respiratory rate 28/min, Heart rate 110/min. Examination reveals intercostal retractions, dimin-
ished air entry on both lungs. What is the most likely diagnosis?
Bronchial asthma
Bronchiolitis
epiglottitis
foreign body aspiration
pneumonia

79) A 24-year-old PG pregnant at 9 weeks complains of mild vaginal spotting one day after effort.
Ultrasound scan revealed a viable 9 weeks fetus with a small caudal separation of the sac. What
is the appropriate management?
85
Immediate termination of pregnancy.
Transvaginal cervical cerclage
Conservative follow up.
Start tocolytic drugs.
Antiprostaglandin rectally.

80) A 3 years old girl presents to the outpatient clinic with multiple red spots all over the body. Her
mother says that the spots started over the trunk followed by the extremities. She had, runny
nose and mild fever two weeks before. On Examination the girl was pale, temperature was 37.6°
C, and spleen was 4 cm below the costal margin. Which is the most appropriate initial investiga-
tion?
ASOT
blood culture
bone marrow aspirate and biopsy
CBC with direct platelet count
kidney function tests

81) A 9-month-old baby boy has been admitted to the pediatric unit with intermittent colicky abdom-
inal pales with blood-stained stools and mucus in his diaper (red currant jelly stools). On exam-
ination he looks unwell. Abdominal examination shows an empty right iliac fossa with a lump in
the epigastrium. What is the appropriate diagnosis?
Acute ileocolic intussusception.
Caecal volvulus.
Carcinoma of caecum with acute distal small bowel obstruction.
Duodenal atresia.
Paralytic ileus.

82) A 30-year-old primigravida presents at 34 weeks’ gestational age with blood pressure of
170/100 mmHg, headache, epigastric pain, visual abnormalities and 3+ proteinuria. Biophysical
profile of the fetus is 8/8. Which one of the following is the immediate response?
Start antihypertensives.
Immediate refer for emergent termination
Give betaciomethasone to induce fetal lung maturity.
Perform an amniocentesis to assess fetal lung maturity.
Repeat the biophysical profile daily.

83) A 9 months old infant presents to the GP with delayed sitting and standing. Examination reveals
frontal bossing, anterior fontanel 3 fingers broad, bow legs and pigeon shaped chest. What is the
most likely elevated laboratory test?
alkaline phosphatase
calcuim
magnesuim
phosphorus
vitamin D
86
84) A 35-year-old Para 2 was diagnosed to have invasive mole (persistent trophoblastic disease) of
low- risk prognostic score. What is the appropriate management next to evacuation of her molar
pregnancy?
Total abdominal hysterectomy.
Chemotherapy with one or more anti-cancer drugs.
Endometrial biopsy two weeks after evacuation.
No further management is required.
Follow up with serum B-hCG level

85) A 30-year-old Para 3 asked her doctor about her chances of getting a boy after having 3 girls.
What is the proper explanation of her fetal sex determinant?
She has a dominant X-chromosome.
Sperm carries only X-chromosome.
She has a familial tendency to get females
Incapacitated Y-carrying spermatozoa.
There is always a chance to get a boy.

86) A 55-year-old man comes to the office because of a one-day history of sudden onset of severe
pain in his low back that radiates down his left leg, and a “pins and needles feeling in his left
heel. The pain increases if he remains in a static position for longer than 10 minutes. He has had
difficulty sleeping due to pain. He has had 3 episodes of moderate low back pain that did not ra-
diate during the past 4 months days//weeks//months//years. During past episodes, nonsteroidal
anti- inflammatory drugs and muscle relaxants provided complete relief of his symptoms. During
this current episode he has no other history of serious illness and takes no routine medication.
His temperature is 370C, pulse is 85/min and regular, respirations are 16/min, and blood pres-
sure is 140/100 mm Hg. Cardiopulmonary and abdominal examinations show no abnormalities.
Bladder and bowel functions are normal. Sensation in the groin is intact. Muscle strength is nor-
mal in the lower extremities. Deep tendon reflexes are normal bilaterally. Flexion of the left lower
extremity is limited to 30 degrees due to pain. Which of the following is most likely cause of this
patient’s symptoms diagnosis?
herniated disc
myocitis
spinal stenosis
spinal tumor
spondylolisthesis

87) A 33-year-old gravida 2 Para 1 (stillborn infant). She had a screening test for diabetes mellitus at
10 weeks’ pregnancy. One-hour post-prandial glucose test is 145 mg/dl. What should be includ-
ed in the follow-up of this patient?
No further testing and no need for treatment
Serial 3D ultrasound.
Standard glucose tolerance test.
Serial follow up with domiciliary glucose urine testing.
Serial follow up with domiciliary blood sugar testing.

88) A one-year old boy is brought to the clinic by his mother for routine well child examination. He
87
is exclusively breast-fed till now. The mother says she tried to introduce cereals, but he refused.
On examination the child is pale with no organomegaly. The GP requests a blood picture which
shows a Hg 8.7 gm/dl (N: 14-17 gm/dl) and HCT 24%. What is the most likely diagnosis?
thalassemia
iron deficiency anemia
pernicious anemia
sideroblastic anemia
spherocytosis

89) A newly married 30-year-old lady was informed by her gynecologist about the benefit of precon-
ceptional folic acid supplementation. She asked about duration of its use after being pregnant.
What would be the appropriate recommendation?
To be discontinued.
Till end of 12 weeks.
Till end of 20 weeks.
Till end of 36 weeks.
Throughout the entire pregnancy

90) A 52-year-old woman comes to the emergency department because of a 3-week history of short-
ness of breath and nonproductive cough. The shortness of breath has progressively worsened
during the past 2 days. She also has a 3-week history of low back mild pain. Ibuprofen has pro-
vided minimal relief of her pain. She has no history of serious illness and takes no routine medi-
cations). She began smoking at age 32 and smoked one pack of cigarettes daily. Her temperature
is 37.0°C (98.6°F), pulse is 105/min, respirations are 26/min, and blood pressure is 180/118
mm Hg. Pulse oximetry on room air shows an oxygen saturation of 88%. Physical examination
shows jugular various distention to 3 cm above the sternal angle. Auscultation of the lungs
discloses bilateral wheezing and basilar crackles. Cardiac examination discloses an S3. There is
bilateral edema of the lower extremities. Chest x-ray shows cardiomegaly, bilateral infiltrates,
and bilateral pleural effusions at the lung base. Which of the following is the most appropriate
initial pharmacotherapy?
intravenous chlorothiazide
intravenous digoxin
intravenous dobutamine
intravenous furosemide
oral carvedilol

91) A 50-year-old man is seen in the emergency center for abdominal pain. A provisional diagnosis of
acute cholecystitis is made. Which of the following findings is most consistent with this diagno-
sis?
Fever, intermittent right upper quadrant pain, and jaundice.
Persistent right upper quadrant pain, fever and leucocytosis.
Epigastric and back pain.
Painless jaundice with palpable gall bladder.
Generalized abdominal pain and tenderness.

92) A health full term breast-fed infant presents with jaundice at 36 hours of age. Total serum biliru-
88
bin is 20 mg/dl, direct bilirubin 0.9 mg/dl and reticulocytic count is 10%. The baby’s blood group
is A negative and mother’s blood group O negative. Which is the most likely diagnosis?
ABO incompatibility
breast milk-associated jaundice
Criggler Najjar Syndrome
hereditary spherocytosis
Rhesus hemolytic disease

93) Which one of the following attitudes are accepted in Physician- physician relationship?
receiving fees for patient referral
taking commission from lab
taking patients from colleagues
treating colleagues with reduced fees
treating colleagues without fees

94) A 60-year-old woman comes to the emergency department 2 hours after she vomited coffee
ground-like material. She has no history of similar symptoms. She has osteoarthritis of both hips
and hypertension. She is on daily non-steroidal, anti-inflammatory tablet and Enalapril tablet.
She started smoking at age 30 and smoked one pack of cigarettes daily. She is 160 cm tall and
weighs 85 kg. Her temperature is 36.9°C, pulse is 110/min, respirations are 20/min, and blood
pressure is 120/70 mm Hg Physical examination shows no other abnormalities. Nasogastric
aspiration yields 200 mL of coffee- ground material. A 0.9% saline stomach lavage eventually
cleared the aspirate. Which of the following is the most likely diagnosis?
esophageal varices
esophagitis
gastric cancer
Acute gastric erosion
Mallory-Weiss syndrome

95) A 36-year-old man comes to the office because of a 3-day history of burning pain with urination
and a 1-day history of fever [temperatures to 39.5°C chills, and severe pain in his back. He has
type 1 diabetes mellitus treated with insulin therapy. His mother had type 1//2 diabetes mellitus
and died 3 years ago at age 62 years because of acute renal failure. He has smoked one pack(s) of
cigarettes daily for 20 years. His temperature is 38.5°C (101.3°F), pulse is 100/min, respirations
are 18 min, and blood pressure is 150/70 mm Hg. Physical examination shows mild tenderness
(pain) to deep palpation of the left loin. Urinalysis shows: Which of the following is the most like-
ly diagnosis?
cystitis
prostatitis
pyelonephritis
nephrotic syndrome
glomerulonephritis

96) A two-year-old boy is rushed by his mother to the Emergency Room with difficult breathing and
coughing for the last 2 days. Temperature is 37° C, Respiratory Rate 34 /min, HR 100/min Exam-
ination reveals diminished air entry and wheezes on the right side of the chest only. Chest X-ray
89
reveals overdistended right lung, especially in expiratory film. What is the most appropriate next
management step?
administer steroids
perform chest physiotherapy
perform bronchoscopy
use a nebulizer with ipratropium
use a nebulizer with beta 2 agonist

97) 44-year-old woman comes to the emergency department because of a 1-day history of severe
pain in her upper abdomen that radiates to her back, nausea, and vomiting. She has had 3 epi-
sodes of emesis during this time. She has no history of serious illness and takes no medications.
She does not smoke cigarettes. Her temperature is 37.0°C (98.6°F), pulse is 111/min, respira-
tions are 22/min and shallow, and blood pressure is 118/72 mm Hg. Pulse oximetry on room air
shows an oxygen saturation of 97%. The abdomen is soft and mildly distended. There is sever
right upper quadrant and epigastric tenderness (pain) to palpation. No organomegaly is noted.
Which of the following is most likely to confirm the suspected diagnosis?
CT scan of the abdomen with contrast
liver isotope scan
transesophageal echocardiography
ultrasonography of the abdomen
upper gastrointestinal endoscopy

98) If you are employed as a doctor in the emergency room: What is your recommendation to the
nurses on when to revise their hand hygiene?
After contact with a patient
Before performing an aseptic task eg. insertion of an LV cannula
Before contact with blood, body fluids or contaminated surfaces
If hands will be moving from a clean body site to a contaminated body site during patient care
Before removal of personal protective equipment (PPE).

99) What is the definition of Marasmus in relation to body weight according to Wellcome classifica-
tion?
between 60-80% of expected weight for age without edema
between 60-80% of expected weight for age with edema
above 80% without edema
under 60% of expected weight for age with edema
under 60% of expected weight for age without edema

100) A 74-year-old woman with a BMI of 36 presents with a painful lump around her umbilicus. She
has had it for several years and it has increased in size recently. Examination reveals a large,
tender, hard, lump over the umbilicus with bluish discoloration and no cough impulse. Her labo-
ratory findings showed White blood cells 16x103/cc (5-11x103/cc). Hb. 9.8 mg/dL (11-13 mg/dl)
and ALT 100 IU/L (12-40 IU/L What is the appropriate diagnosis?”
Paralytic lleus.
Sigmoid volvulus
90
Strangulated femoral hernia.
Strangulated umbilical hernia.
Umbilical granuloma

91

You might also like